Practice Questions

Lakukan tugas rumah & ujian kamu dengan baik sekarang menggunakan Quizwiz!

A patient presents with eye pain and blurred vision. Snellen testing reveals vision of 20/200 in the affected eye and 20/20 in the unaffected eye. Fluorescein staining reveals the presence of a dendritic ulcer. Which of the following is the most likely diagnosis? A. Viral keratitis B. Fungal corneal ulcer C. Acanthamoeba keratitis D. Bacterial corneal ulcer

A. Question 1 Explanation: Herpes Simplex virus is a common cause of dendritic ulceration noted on fluorescein staining RF = CONTACT LENSES

An 85-year-old nursing home patient was seen in a local physician's office during the day for a corneal abrasion. The patient had antibiotic drops instilled, and the eye was patched. At 10: 00 p.m., the nursing staff calls reporting the patient is very confused. The most appropriate action is to A. Remove the eye patch B. Prescribe haloperidol C. Have the patient taken to the emergency room D. Reassure the nursing staff and see the patient the next day

A. Question 1 Explanation: Sensory deprivation, such as patching an elderly patient's eyes, may lead to an acute case of delirium. Even small alterations in the elderly patient's environment can lead to confusion. In cases of corneal abrasions, an elderly patient should receive topical ophthalmic antibiotics. Although eye patching traditionally has been recommended in the treatment of corneal abrasions, multiple well-designed studies show that patching does not help and may hinder.

A 23 year-old sexually active female presents with a 4 day history of painless bilateral eye exudates which she describes as copious. Visual acuity is 20/20, generalized conjunctival inflammation with sparing of the cornea is noted on physical examination. Gram stain of the exudate reveals gram negative diplococci. Appropriate management of this case is A. ceftriaxone (Rocephin) B. polymyxin ophthalmic drops (Aerosporin) C. ciprofloxacin (Cipro) D. doxycycline (Doryx)

A. Question 8 Explanation: With sparing of the cornea, as in this case, a single 1 gram IM dose of ceftriaxone is sufficient treatment for ophthalmic gonorrhea. If the cornea is involved, 5 days of IM ceftriaxone would be required.

A 60-year-old female presents with painless, atraumatic vision loss in her left-eye since last evening. The patient states that she initially experienced flashes of light in her eye and floaters and that these symptoms progressed to vision loss. At first, she lost vision in the periphery, like "a curtain coming down" over her eye. The patient's visual loss then progressed centrally. Fundoscopic exam reveals a postiive Schaffer's sign. What is the most likely diagnosis?

A. Retinal detachment

A 9-year-old male is brought into the office for "pink eye". He developed the symptoms 3 days ago while vacationing with his grandparents. The child reports symptoms began a day after going to the local water park. The child has had no fevers, upper respiratory infection symptoms or visual changes. He complains of watery discharge and irritation. Physical exam reveals bilaterally injected conjunctiva and copious watery discharge. EOM's are intact and painless with movement, PERRLA and visual acuity is within normal limits. Bilateral preauricular adenopathy is present as well. The remainder of the HEENT exam is unremarkable. What is the most likely diagnosis? A. Chalazion B. Viral conjunctivitis C. Bacterial conjuncitvitis D. Dacrocystitis

B.

Which of the following is a potential complication of a traumatic hyphema? A. retinal detachment B. glaucoma C. cataract formation D. chronic conjunctivitis

B. Question 1 Explanation: If the trabecular network becomes obstructed from the hyphema then glaucoma may occur.

Which of the following findings is most consistent with cataracts? A. conjunctival injection B. poorly visualized optic disc C. central visual field loss D. arcus senilis

B. Question 2 Explanation: Cataracts are caused by opacification of the crystalline lens, and this decreases the amount of light that enters the eye. It is difficult to see through the lens from either direction, and thus, the optic disc is poorly visualized on examination.

A patient is evaluated in the office with a red eye. The patient awoke with redness and a watery discharge from the eye. The eyelids were not matted together. Examination reveals a palpable preauricular node. Which of the following is the most likely diagnosis? A. bacterial conjunctivitis B. viral conjunctivitis C. allergic conjunctivitis D. gonococcal conjunctivitis

B. Question 7 Explanation: Viral conjunctivitis is associated with copious watery discharge and preauricular adenopathy.

A 59 year-old male complains of "flashing lights behind my eye" followed by sudden loss of vision, stating that it was "like a curtain across my eye." He denies trauma. He takes Glucophage for his diabetes mellitus and atenolol for his hypertension. He has no other complaints. On funduscopic exam, the retina appears to be out of focus. Which of the following is the most likely diagnosis? A. Central retinal vein occlusion B. Retinal artery occlusion C. Retinal detachment D. Hyphema

C. Question 1 Explanation: Patients with retinal detachment frequently complain of flashes of light or floaters that occur during traction on the retina as it detaches. This is followed by loss of vision. In small detachments, the retina may appear out of focus, but with larger detachments, a retinal fold may be identified.

Use of systemic corticosteroids can cause which of the following adverse effects in the eye? A. Cortical blindness B. Optic atrophy C. Glaucoma D. Papilledema

C. Question 2 Explanation: Glaucoma can be caused by the long-term use of steroids.

A 65-year-old woman presents complaining of "yellow vision." The patient states she has no pain associated with her change in vision. The physician assistant noticed a small central lens opacity on her last visit that seems to have enlarged. Examination reveals a diminished red reflex. The most likely diagnosis is which of the following? A Cataract B Glaucoma C Keratitis D Pterygium

Correct Answer ( A ) Explanation: A cataract is an opacity of the lens that decreases visual acuity. It is caused by oxidative damage to the lens, leading to increased deposition of insoluble proteins in otherwise transparent tissue. Risk factors for developing a cataract include smoking, diabetes, and corticosteroid therapy. Classically, cataracts present with painless blurred vision often with a yellow tint. Symptoms are progressive developing over months to years. Lens opacities can be grossly visible or seen as a diminished red reflex. Diagnosed is made with slit-lamp biomicroscopy during ophthalmologic exam. Definitive diagnosis is surgical removal of the cataract and replacement of the intraocular lens. Glaucoma (B) is characterized by increased intraocular pressure, leading to irreversible damage to the optic nerve. It presents as progressive peripheral vision loss leading to tunnel vision. Patients may also complain of "halos around lights." Keratitis (C) is inflammation of the cornea. Common symptoms include pain, impaired eyesight, photophobia, red eye and a "gritty" sensation. Pterygium (D) is a benign growth of the conjunctiva that commonly grows from the nasal side of the sclera. In advanced cases the pterygium can affect vision as it invades the cornea with the potential of obscuring the optical center of the cornea and inducing astigmatism and corneal scarring. Question: What is the name of the benign growth that presents as a yellow patch or bump on the conjunctiva but does not invade the cornea? Answer: Pinguecula. Cataract Opacity of lens Painless blurred vision Treatment is surgical

A 71-year-old woman presents for "pink eye." She is rubbing her eyes, and complains of itching. Both eyes appear irritated and red-rimmed with clear scales adherent to her lashes. The meibomian glands appear inflamed. There are no changes in vision, and she denies any exposure to allergens or chemicals. She states this happened in the past. Which of the following is the most likely diagnosis? A Blepharitis B Chalazion C Dacryocystitis D Keratoconjunctivitis sicca

Correct Answer ( A ) Explanation: Blepharitis is the most likely diagnosis, based on her symptoms of eye burning and itching, as well as a physical exam consistent with meibomian gland swelling and scaling of the lid margins. For many patients, blepharitis becomes a chronic condition with recurrent acute flares. However, acute flares can be minimized with adherence to a regimen for eye cleanliness, which consists of removal of eyelid scales daily with a warm wash cloth and baby shampoo. When flares of blepharitis are limited to the lashes and margins (anterior blepharitis), patients may benefit from topical application of erythromycin or bacitracin to the margins. However, in patients with inflammation of the meibomian glands and orifices (posterior blepharitis), particularly when there are also signs of inflammation of the conjunctiva or cornea, oral antibiotics are indicated. This may need to be prescribed as a long-term therapy, such as daily treatment with doxycycline or erythromycin. A chalazion (B) would present as a localized area of firm swelling where granulomatous inflammation of a meibomian gland has occurred. While a chalazion may commonly form after patients have had a tender internal hordeolum ("stye"), a chalazion itself should not be painful or cause itching. Dacryocystitis (C), infection of the lacrimal sac, typical results in pain, swelling, and erythema overlying the tear sac particularly. This would be less likely to cause itching or scaling of the lid margins. Keratoconjunctivitis sicca (D), which is essentially "dry eye syndrome," would be unlikely to yield the symptoms seen in this patient. Rather, patients are more likely to complain of dry eyes, redness, and a foreign body sensation. Question: What might be a typical appearance of the tears in a patient with blepharitis? Answer: Tears in patients with blepharitis may appear frothy with a greasy sheen. Blepharitis Patient will be complaining of eyelid changes and eyelash flaking PE will show crusting, scaling, and red-rimming of the eyelid Diagnosis is made by slit-lamp examination Most commonly caused by dysfunctional meibomian gland Treatment is warm compresses, irrigation, lid massage, and topical antibiotics for flare-ups Comments: associated with seborrhea and rosacea

A 65-year-old woman complains of acute pain in the left eye and vomiting after walking into a movie theater. On examination, her cornea is cloudy. She has photophobia and a fixed pupil. She reports an allergy to sulfa. What is the most appropriate action? A Administration of acetazolamide B Measurement of intraocular pressure C Topical mydriatic agent D Ultrasound of the eye

Correct Answer ( B ) Explanation: Acute angle-closure glaucoma occurs classically when patients move into a situation with lower light requiring dilation of the pupil and then obstruction of aqueous humor outflow track. Patients experience the abrupt onset of symptoms including eye pain, headache, vomiting, and blurred vision. Patients may also describe seeing a halo around lights. On examination, the conjunctiva is injected with a cloudy appearance to the cornea. The pupil is typically mid-sized or dilated and either fixed or sluggishly reactive. One must suspect the diagnosis clinically and then measure and document the intraocular pressure. Goals of treatment include rapid reduction of intraocular pressure through both topical and intravenous agents. Topical agents include beta-blockers (e.g. timolol), miotic agents (pilocarpine), and steroids. In addition, acetazolamide (carbonic anhydrase inhibitor) is commonly used to decrease aqueous humor production. Mannitol (an osmotic diuretic) can be used as an intravenous agent for severe cases. Definitive correction requires surgery. Administration of acetazolamide (A) is part of the standard treatment for acute angle-closure glaucoma, however, this patient reports a sulfa allergy and therefore is contraindicated. A topical mydriatic agent (C) would further dilate the pupil and worsen the symptoms. Pilocarpine is administered as a miotic agent to constrict the pupil and hopefully allow flow of the aqueous humor. An ultrasound of the eye (D) will not be helpful in the diagnosis of this condition. Ultrasound is routine in the evaluation of patients for retinal or vitreous detachment and also as a surrogate for intracranial pressure through measurement of the optic nerve diameter. Question: What is normal intraocular pressure? Answer: Less than 21 mm Hg. Acute Angle-Closure Glaucoma Patient will be entering a dark room or movie theater Complaining of acute unilateral painful vision loss, vomiting, and seeing halos around lights PE will show cloudy cornea and fixed mid-dilated pupil Testing will show IOP ( > 21 mmHg) Treatment is topical beta-blockers, carbonic anhydrase inhibitors, mioticsIridotomy is definitive treatment, but not used for initial management

Which of the following conditions is associated with a "blood and thunder" retinal appearance? A Central retinal artery occlusion B Central retinal vein occlusion C Macular degeneration D Retinal detachment

Correct Answer ( B ) Explanation: Central retinal vein occlusion most commonly occurs as a result of a thrombotic event. Hyperviscosity states such as polycythemia or leukemia can predispose a patient to this condition due to slowing of blood flow leading to thrombus formation. Other risk factors include diabetes, hyperlipidemia, glaucoma, and atherosclerotic disease. Atherosclerosis of the retinal artery causes the artery to become very rigid. Because the retinal artery and vein are located in the same narrow anatomical space, rigidity of the artery causes impingement on the weak venous walls. This can create turbulence of the venous blood or endothelial damage leading to thrombus formation. Central retinal vein occlusion can be divided into two types; ischemic and nonischemic. Nonischemic is the more common and less severe form for central retinal vein occlusion but may progress to an ischemic form. Those with the nonischemic form may present with an insidious onset of minor alterations in vision that may come and go. Disease progression or presentation of ischemic changes include severely diminished or absent vision, photophobia, and pain or redness of the eye. The classic "blood and thunder" retinal appearance represents retinal hemorrhages covering the entire retina. Many patients will have retinal hemorrhages that are less diffuse. Other fundoscopic findings include dilated, tortuous veins, optic disc edema or cotton wool spots. Severity of disease and prognosis is dependent upon how much of the retina is devascularized. There is no known preventative or therapeutic treatment of this condition. Minimizing risk factors related to comorbidities is imperative. Central retinal artery occlusion (A) is considered to be an ophthalmic emergency and carries a poor prognosis. It presents as a painless, sudden onset of unilateral vision loss most commonly caused by an embolic or thrombotic event. It is associated classically with retinal edema and perifoveal atrophy represented as a cherry-red spot. Macular degeneration (C) causes an insidious onset of diminished central vision typically occurring after age 50. Metamorphopsia, the appearance of wavy or distorted vision, is also characteristic of this disorder. On fundoscopic exam, drusen deposits may be seen. Retinal detachment (D) is characterized by the onset of painless blurred or dark vision that often progresses to total blindness. It is classically described as a curtain being drawn down over the field of vision from top to bottom. Fundoscopic exam may show ridges of the displaced retina as it flaps in the vitreous humor. Question: What tool may be used to assess for metamorphopsia seen with macular degeneration? Answer: Amsler grid. Central Retinal Vein Occlusion Patient with a history of HTN or DM Complaining of varying degree of sudden painless monocular vision loss Fundoscopic exam will show "blood and thunder," tortuous, and dilated retinal veins Management includes ophthalmology consultation

A 35-year-old man presents with right eye pain after drilling at his construction site five hours earlier. You note a metallic foreign body at the 3 o'clock position on the cornea and remove it using a 27-gauge needle. A rust ring is visible on the cornea. When is the most appropriate time to attempt rust ring removal? A 1 hour after development of the rust ring B 24 to 48 hours after development of the rust ring C 4 to 6 hours after development of the rust ring D 5 to 7 days after development of the rust ring

Correct Answer ( B ) Explanation: Metallic foreign bodies embedded in the cornea develop rust rings due to oxidation of the iron. Rust rings can develop within hours of metallic foreign body contact with the cornea. Over 24-48 hours, the rust ring will oxidize and kill surrounding epithelium. At this time, the rust ring will be soft and easily removed in one solid plug. The surrounding epithelium may not be easily removed (A and C) until 24 hours after the rust ring develops. After 5-7 days (D), the rust ring and surrounding epithelium is at risk for infection and scarring. Alternatively, the rust ring may be removed at the same time as the foreign body removal, either with an eye spud or with a rotating bur if the emergency physician is comfortable with this procedure. Question: Can an MRI be used to identify intraocular foreign bodies? Answer: If the foreign body is metallic, an MRI is contraindicated because it can dislodge the foreign body and damage surrounding tissue. Corneal Abrasion Patient will be complaining of pain, photophobia, tearing, or foreign body sensation Diagnosis is made by fluorescein stain Treatment: Most are self-limiting,anti pseudomonal for contact lens wearers

A 70-year-old man, with a past medical history of type 2 diabetes, presents for a routine eye exam. He has noticed a gradual loss of peripheral vision since his eye exam last year. On examination, he has diminished visual fields, elevated intraocular pressure, and symmetrically enlarged cup-to-disk ratio bilaterally. What is the most likely diagnosis? A Diabetic retinopathy B Macular degeneration C Open-angle glaucoma D Presbyopia

Correct Answer ( C ) Explanation: Open-angle glaucoma is the second leading cause of blindness in the United States behind diabetic retinopathy. It is characterized by optic neuropathy and can be associated with increased intraocular pressure. Risk factors include family history, advanced age, and diabetes mellitus. African-Americans are more likely to develop glaucoma than other racial groups. The most common symptom is a gradual loss of peripheral vision, but often patients are asymptomatic. Physical exam will reveal an enlarged cup-to-disk ratio in one or both eyes and may reveal diminished visual fields. Intraocular eye pressure (IOP) may be normal or increased. Even if the IOP is within the normal range, treatment is still aimed at decreasing the pressure to slow the progression of damage. Topical eye drops that decrease aqueous humor production or increase outflow are first-line for treatment. Diabetic retinopathy (A) can also be largely asymptomatic or can manifest as slow decrease in visual acuity. Fundoscopic exam may reveal intraretinal hemorrhages, cotton-wool spots, neovascularization, and macular edema. Macular degeneration (B) will present with blurring of the central vision, central blind spots, or intermittent shimmering lights in the visual field. Fundoscopic exam will reveal drusen or areas of depigmentation on the macula. Presbyopia (D) is a normal decrease in near-vision as a part of aging. The fundoscopic exam should be normal. Acute Angle-Closure Glaucoma Patient will be entering a dark room or movie theater Complaining of acute unilateral painful vision loss, vomiting, and seeing halos around lights PE will show cloudy cornea and fixed mid-dilated pupil Testing will show IOP ( > 21 mmHg) Treatment is topical beta-blockers, carbonic anhydrase inhibitors, mioticsIridotomy is definitive treatment, but not used for initial management

A 51-year-old man walks into a movie theater and experiences acute onset of right eye pain associated with nausea, vomiting, and cloudy vision. Which of the following is expected during the ophthalmologic exam? A Cherry-red spot in the macular area B Deep anterior chamber C Intraocular pressure >21 mm Hg D Miotic pupil

Correct Answer ( C ) Explanation: The patient has acute angle-closure glaucoma. This condition results in optic nerve damage from increased intraocular pressure. In patients with a narrow anterior chamber angle, reduced illumination (like entering a dark movie theater) causes mydriasis. Subsequently, folds of the peripheral iris can block the angle, which prevents aqueous humor outflow. This leads to a rapid elevation of intraocular pressure causing ocular pain, a hazy cornea, ciliary flush, a firm globe, and optic nerve damage if the pressure is not promptly relieved. An intraocular pressure >21 mm Hg is considered elevated. Pressures can elevate quickly to >60 mm Hg. The higher the pressure, the quicker damage occurs to the optic nerve and the poorer the prognosis. The diagnosis is often delayed due to the associated symptoms of nausea, vomiting, and abdominal pain. Treatment involves reducing aqueous humor production with IV acetazolamide, topical beta-blockers (timolol), and topical alpha-agonists (apraclonidine). Topical miotic agents (pilocarpine) are used to reverse the angle closure. Topical steroids help to reduce inflammation. Hyperosmotic agents (mannitol, glycerol) can also be administered for further reduction in intraocular pressure. The funduscopic finding of a pale retina with a cherry-red spot in the macular area (A) is consistent with central retinal artery occlusion. This condition is associated with sudden unilateral vision loss that is painless. It is caused by a thrombotic plaque or embolus of the central retinal artery. A deep anterior chamber (B) is protective against acute angle-closure glaucoma. Individuals with a narrow chamber are at increased risk. The pupil in acute angle-closure glaucoma is most commonly fixed and mid-dilated, rather than miotic (D). Miotic pupils are associated with opiate use, cholinergic toxicity, and pontine strokes. Question: What is Adie's pupil? Answer: A pupil with parasympathetic denervation that constricts poorly to light but reacts better to accommodation. Gives the appearance of anisocoria. Acute Angle-Closure Glaucoma Patient will be entering a dark room or movie theater Complaining of acute unilateral painful vision loss, vomiting, and seeing halos around lights PE will show cloudy cornea and fixed mid-dilated pupil Testing will show IOP ( > 21 mmHg) Treatment is topical beta-blockers, carbonic anhydrase inhibitors, mioticsIridotomy is definitive treatment, but not used for initial management

A 65-year-old man complains of sudden pain and vision loss in his left eye about 20 minutes after entering a movie theater. Which of the following is the most appropriate treatment for his condition? A Cyclopentolate B Digital massage of the globe C High-dose steroid D Topical beta-blocker

Correct Answer ( D ) Explanation: Acute angle glaucoma can be triggered by dim light (such as going from bright light outside to a darkened theater), which dilates the pupil leading to occlusion of the trabecular meshwork and obstruction to aqueous humor outflow resulting in increased intraocular pressure. Treatment is aimed at rapid reduction of intraocular pressure by inhibiting production of aqueous humor and improving outflow. This can be achieved with a topical beta-blocking agent such as timolol. Additional medications that may be used include alpha-agonists (brimonidine), prostaglandin analogues (latanoprost), and pilocarpine (topical miotic). Adjunctive therapy includes carbonic anhydrase inhibitors (acetazolamide) and osmotic diuretics (mannitol). Corneal indentation is reserved for intraocular pressures >50 mm Hg or when other agents are ineffective. Cyclopentolate (A) is a mydriatic agent that can induce and worsen acute angle glaucoma. As such, it should be avoided in all patients at risk for this condition. Digital massage of the globe (B) is a technique used in patients with central retinal artery occlusion to help dislodge the embolus. High-dose steroids (C) are used in patients with vision loss due to temporal arteritis. Acute Angle-Closure Glaucoma Patient will be entering a dark room or movie theater Complaining of acute unilateral painful vision loss, vomiting, and seeing halos around lights PE will show cloudy cornea and fixed mid-dilated pupil Testing will show IOP ( > 21 mmHg) Treatment is topical beta-blockers, carbonic anhydrase inhibitors, mioticsIridotomy is definitive treatment, but not used for initial management

A mother has noticed a "cross-eye" in her 3-month-old boy. Examination reveals no gross external deformities. Pupillary light reflexes are intact. The eyelids, sclerae and pupils are normal appearing. A cover-uncover test reveals a left eye esotropia. Which of the following is the most likely diagnosis? A Amblyopia B Ectropion C Leukocoria D Strabismus

D Strabismus, also known as "cross-eye" or "wall-eye", is a condition of eyeball misalignment that results in the inability of both eyes to fixate on the same point in space. This results in poor depth perception and diplopia. It occurs in 5 of every 100 US citizens, and is one of the most common eye disorders of infants and children. It is mostly due to extraoccular muscle imbalance or central nervous system abnormalities, such as cranial nerve palsies, Brown's syndrome and Kearns-Sayre syndrome, although the exact mechanism of disease is largely unknown. The eyes, or one eye, may point in (esotropia), out (exotropia), up (hypertropia) or down (hypotropia). A cover-uncover test is required in making the diagnosis. The current recommendation is to screen for strabismus at every well child check. Early diagnosis and treatment offer the best chance at normal binocular vision. Left untreated, strabismus can lead to amblyopia (A), also known as "lazy eye". In this condition, decreased acuity is not due to any structural anomaly or visual pathway abnormality, but is a result of the brain ignoring the visual input from the strabismic eye in an attempt to limit diplopia and poor depth perception. Half of the cases of ambylopia are caused by strabismus. Ectropion (B), the outward turning of the lower eyelid, is mainly due to aging degeneration of the supporting connective tissues. A combination of an abnormal pupillary light reflex and a whitish-appearing pupil is called leukocoria (C), and can be caused by retinoblastoma, cataract, vitreous opacity or corneal opacity. Question: Which cranial nerve palsies are associated with strabismus? Answer: The nerves which control the extraoccular muscles: third (occulomotor), fourth (trochlear) and sixth (abducens). Strabismus Esotropia: in Exotropia: out Cover-uncover exam Light reflex test

A 66 year-old male presents complaining of 6 month history of progressive blurred vision without associated pain. On examination there is no erythema or injection of the sclera. On funduscopic examination there is an absent red reflex and a cloudy lens. Which of the following is the most likely diagnosis? A. Retinal detachment B. Chronic glaucoma C. Age-related macular degeneration D. Cataract

D. Question 3 Explanation: Cataracts present with blurred vision that progress over months to years. On examination the red reflex becomes increasingly difficult to visualize until it is finally absent and the pupil is white.

Which of the following does the macula provide? A. Night vision B. Color vision C. Peripheral vision D. Central vision acuity

D. Question 1 Explanation: The macula is responsible for central visual acuity

You are evaluating a 67-year-old Asian male in the emergency room for acute onset right eye pain. He states he was at the evening premier of a newly released movie when the pain started. He had acute, profound visual loss in the affected eye. The pain was intense enough for him to leave the theatre before the movie's conclusion, and present to your location. On examination, the eye appears injected (red) and the cornea appears hazy. His pupils are 6 mm on the affected side and 3 mm on the unaffected side. They respond to light on the unaffected side but not on the affected side. On palpation, the globe feels tense. What history question is most relevant to support the diagnosis? A. Contact lens use B. Past sexual contacts C. Recent URI symptoms D Visualizing halos around street lights

D. Question 5 Explanation: Primary acute angle-closure glaucoma occurs only with closure of a pre existing narrow anterior chamber angle found in older age groups, hyperopes, inuites, and Asians. Angle closure may be precipitated by pupillary dilation and thus can occur from sitting in a darkened room, at times of stress or, rarely, from pharmacologic mydriasis. The symptoms given are classic for acute angle-closure glaucoma (older age group, Asian, rapid onset of severe pain and profound visual loss with halos around light, red eye, steamy cornea, dilated pupil, hard eye to palpation).

A 9-year-old presents complaining of a swollen red eye. The nurse notes that the patient is febrile to 103ºF. On exam you note the findings seen above. The patient has pain with eye movement and you note an afferent pupillary defect. Which of the following treatment regiments should be immediately instituted? FLIP FOR IMAGE DONT CHEAT A Amoxicillin-Clavulanate B Bacitracin ophthalmic ointment C Cefazolin and erythromycin D Ceftriaxone and vancomycin

D. Cetriaxone and vancomycin Orbital cellulitis is a life-threatening infection of the tissues posterior to the orbital septum. It occurs most commonly in children. The causative organisms include S. aureus, S. pneumoniae, H. Influenzae, and S. pyogenes. The infection is either spread locally from a dental infection, sinus infection, dacryocystitis, or trauma. Patients present with very painful erythematous swelling of the eyelid, conjunctivitis, proptosis, opthalmoplegia, and fever. There may be blurry vision and pain with eye movements. An afferent pupillary defect is often present. Patients should be started on broad-spectrum intravenous antibiotics that are antistaphylococcal and antistreptococcal such as ceftriaxone. Vancomycin is added to cover MRSA. A CT scan can be obtained to evaluate for postseptal extension or abscess. If there is concern for associated cavernous sinus thrombosis, an MRI with venous phase is indicated. Ophthalmology and ENT consultation should also be obtained. Amoxicillian-Clauvnate (A) is the treatment of choice for preseptal cellulitis. Preseptal cellulitis is an infection of the subcutaneous tissues anterior to the orbital septum. It can progress to orbital cellulitis. Bacitracin ophthalmic ointment (B) is used to treat simple conjunctivitis, not orobital cellulitis. Although cefazolin (first generation cephalosporin) and erythromycin (C) are antistaphylococcal and antistreptococcal, they are not broad-spectrum enough and do not cover for MRSA Question: What are complications of orbital cellulitis? Answer: Blindness, meningitis, septic cavernous thrombosis. Orbital Cellulitis Patient will be complaining of swelling around the eye PE will show decreased extraocular movement, pain with movement of the eye, and proptosis Diagnosis is made clinically Most commonly caused by S. aureus Treatment is ophthalmology evaluation, antibiotics (Proptosis / pain / fever / HA / increased IOP / restricted or painful eye movement)

Picture showing normal retina / dry / wet

Drusen = DRY Wet = Neovascularization

The best way to differentiate between a Chalazion and a Hordeolum is A. The presence of pus B. hordeolum only appear on the lower eyelid C. pain D. visual changes E. both A and C

E Question 1 Explanation: Chalazion are relatively painless lesions (in comparison to a hordeolum which is a "hot", painful lesion). They are characterized by their insidious onset with minimal irritation. A chalazion is noninfectious obstruction of a meibomian gland causing extravasation of irritating lipid material in the eyelid soft tissues with focal secondary granulomatous inflammation. A hordeolum (stye) is an acute, localized swelling of the eyelid that may be external or internal and usually is a pyogenic (typically staphylococcal) infection or abscess.

The treatment of choice for blepharitis includes: A. Antibiotic ointment (eg, bacitracin/polymyxin B, erythromycin, or gentamicin 0.3% qid for 7 to 10 days) B. Warm compresses over the closed eyelid C. Tear supplements during the day D. Gentle cleansing of the eyelid margin 2 times a day with a cotton swab dipped in a dilute solution of baby shampoo (2 to 3 drops in ½ cup of warm water) E. all of the above

E. Chronic disease is treated with tear supplements, warm compresses, and occasionally oral antibiotics (eg, a tetracycline) for meibomian gland dysfunction or with eyelid hygiene and tear supplements for seborrheic blepharitis. Gentle cleansing of the eyelid margin 2 times a day with a cotton swab dipped in a dilute solution of baby shampoo (2 to 3 drops in ½ cup of warm water) Crusyt eyelids in AM, ABx in the event of Staph Infection

Other than age, which of the following are known risk factors for developing cataracts? A. Alcohol B. Tobacco use C. Metabolic syndrome D. Sunlight exposure E. all of the above

E. Question 1 Explanation: Cataracts are the most common cause of vision loss in people over age 40 and is the principal cause of blindness in the world. Cataracts occur with aging. Other risk factors may include the following: Trauma (sometimes causing cataracts years later), Smoking, Alcohol use, Exposure to x-rays, Heat from infrared exposure, Systemic disease (eg, diabetes), Uveitis, Systemic drugs (eg, corticosteroids), Undernutrition, Chronic ultraviolet light exposure.

A 32-year-old woman comes to your office with a 1-week history of bilateral red eyes associated with tearing and crusting, a sore throat with difficulty swallowing, and a cough that was initially nonproductive but has become productive during the past few days. The patient displays significant fatigue and lethargy, is hoarse, and is having great difficulty performing any of her routine daily chores. On physical examination, there is bilateral conjunctival injection. Her visual acuity is normal. There is significant pharyngeal erythema but no exudate. Cervical lymphadenitis is not present. Examination of the chest reveals a few expiratory crackles bilaterally. What is the most likely organism or condition responsible for the constellation of symptoms in this patient? A. endotoxin-producing Staphylococcus B. endotoxin-producing Streptococcus C. exotoxin-producing Staphylococcus D. activation of the autoimmune system E. none of the above

E. Question 2 Explanation: This picture is completely consistent with adenovirus infection and a primary viral conjunctivitis. Viral agents, especially adenovirus, produce signs and symptoms of upper respiratory tract infection, with the red eye being prominent among those symptoms. With adenovirus, there is often associated conjunctival hyperemia, eyelid edema, and a serous or seropurulent discharge. Viral conjunctivitis is self-limited, lasting 1 to 3 weeks. If the conjunctivitis is definitely caused by a virus, no antibiotic treatment is necessary. There is no indication to perform a throat culture or any other test at this time. The only theoretical concerns are the "rales" that are present in both lung bases; you could argue that if the patient is sick enough, a chest radiograph may be indicated.

A 29-year-old female presents to urgent care with the complaint of a painful, warm, red and swollen lump on her right upper eyelid x1 day. SHe denies associated trauma, visual changes, discharge or fevers. On exam you observe a 3mm area of induration, warmth and erythema localized to the lateral upper lid. There is no surrounding periorbital cellulitis. EOM's are intact and painless with movement, visual acuity is grossly normal. based on history and exam findings, what is the next appropriate initial course of action? A. Warm compresses to affected area x4 daily B. Perform I&D w/ cx C. PO abx D. Immediate emergent referral to ophthalmology

A.

Which of the following may precipitate acute angle-closure glaucoma? A. metoclopramide B. timolol C. glyburide D. acetazolamide

A. Question 3 Explanation: Metoclopramide and other drugs with high anticholinergic effects may precipitate acute angle-closure glaucoma from pupillary dilation

A 72-year-old man presents with a painful red eye and visual loss worsening over the last 24 hours. He recently had cataract surgery. Examination of the eye reveals the image above. Which of the following is the most likely? (FLIP FOR IMAGE DONT CHEAT) A Endophthalmitis B HyphemA C Traumatic iritis D Vitreous hemorrhage

A. Endophthalmitis is an infection involving the anterior, posterior and vitreous chambers of the eye. It results from trauma (blunt globe rupture, penetrating injury, foreign bodies) and also iatrogenically after ocular surgery like cataract repair. Patients complain of severe pain in the eye and visual impairment or loss. Examination of the eye reveals decreased visual acuity, injected conjunctiva, chemosis and haziness of the infected chambers. Infections are treated with both systemic and intraocular antibiotics. A hyphema (B) is blood in the anterior chamber usually caused by trauma. When the patient is in an upright position, blood will layer along the inferior aspect of the anterior chamber. As the hyphema increases in size, it elevates intraocular pressure. In some cases admission is warranted for patients with large hyphemas (>50%), decreased vision, sickle cell disease and elevated intraocular pressure. Treatment is aimed at decreasing pressure with topical (beta-blocker, alpha agonist or carbonic anhydrase inhibitors) and systemic therapy (carbonic anhydrase inhibitor, mannitol). Traumatic iritis (C) occurs after blunt trauma in which the iris and ciliary body are inflamed causing ciliary spasm. Patients complain of significant photophobia with significant eye pain. Examination of the eye reveals perilimbal conjunctival injection (also called ciliary flush) and a small, poorly dilating pupil. Photophobia occurs with light shone on both the affected and unaffected eye. On slit lamp, cells (white and red) and flare (protein) are noted in the anterior chamber. Treatment is with a topical cycloplegic agent to minimize spasm. Vitreous hemorrhage (D) occurs as a result of injuries to the retina, uveal tract and their associated vascular structures. Common associated conditions include diabetic retinopathy, retinal vein occlusion and trauma. Patients complain of decreased visual acuity and floaters. The condition is not typically painful. Diagnosis is made with ocular ultrasound showing blood products in the posterior chamber. Endophthalmitis Infection of anterior, posterior, vitreous chambers Trauma intravitrealABX, ophthalmology consultation, admission

Which of the following is a staphylococcal infection characterized by a localized red swollen and acutely tender abscess of the upper or lower eyelid? A. Hordeolum B. Uveitis C. Chalazion D. Dacryocystitis

A. Question 1 Explanation: Hordeolum (stye) is a staphylococcal infection characterized by a localized red swollen and acutely tender abscess of the upper or lower eyelid. Hordeolum are infections of the glands of the eyelid while chalazion is a sterile, chronic inflammation that results from a blocked meibomian gland TX: Hot compresses, if no improvement I&D. If cellulitis present start oral abx

The underlying pathology causing blepharitis is A. A dysfunction in the meibomian glands B. Impaired aqueous outflow through the trabecular meshwork C. Infectious obstruction of the nasolacrimal gland D. Inflammation or infection of the outer membrane of the eyeball and the inner eyelid.

A. Question 2 Explanation: Blepharitis is caused by the oil glands at the base of the eyelashes (meibomian glands) becoming clogged, a bacterial infection, allergies, or other conditions.

Which of the following is not effective in the treatment of chalazion? A. Systemic antibiotics B. Warm compresses C. Incision and curettage D. corticosteroid therapy injection

A. Question 2 Explanation: Hot compresses for 5 to 10 min 2 or 3 times a day can be used to hasten resolution of chalazia. Incision and curettage or intrachalazion corticosteroid therapy (0.05 to 0.2 mL triamcinolone 25 mg/mL) may be indicated if chalazia are large, unsightly, and persist for more than several weeks despite conservative therapy. Unlike Hordeolum, chalazion are not infectious and will not benefit from the use of oral antibiotic therapy, unless there is concern for surrounding infection.

A patient presents with a nontender, painless, nodule involving a meibomian gland. Which of the following is the most likely diagnosis? A. Chalazion B. Dacryocystitis C. Entropion D. Hordeolum

A. Question 3 Explanation: Chalazion is characterized by a hard, nontender swelling on the upper or lower lid with redness and swelling of the adjacent conjunctiva and is due to granulomatous inflammation of a meibomian gland. PAINLESS LID NODULE. (C = Chalazion = Chronic and "cold") Non-infectious (not hot)

A 12 year old present to your office with red eyes, itching and tearing bilaterally. He has a past medical history significant for asthma. As you examine the inner eyelid what finding do you expect to see? A. cobblestone mucosa B. Kayser-Fleischer rings C. mucopurulent discharge D. dendritic ulcerations

A. Question 4 Explanation: A classic finding of allergic conjunctivitis is cobblestone mucosa on the inner/upper eyelid.

A 56 year-old female presents complaining of intense left eye pain associated with unilateral headache, nausea, and colored rings around lights. On examination you note decreased visual acuity, a pupil that is fixed and mid-dilated, and ciliary flushing. Which of the following is the most likely diagnosis? A. Acute glaucoma B. Migraine C. Episcleritis D. Acute uveitis

A. Question 4 Explanation: Acute glaucoma is an ocular emergency that presents as an acutely painful eye and elevated intraocular pressure. Patients typically complain of acute eye pain associated with unilateral headache, nausea/vomiting, cloudy vision, and colored rings around lights. On exam the pupil is fixed and mid dilated with prominent ciliary flush.

A 17-year-old girl comes to your office with a 1-day history of red eye. She describes not being able to open her right eye in the morning because of crusting and discharge. The right eye feels swollen and uncomfortable, although there is no pain. On examination, she has a significant redness and injection of the right bulbar and palpebral conjunctivae. There is a mucopurulent discharge present. No other abnormalities are present on physical examination. Her visual acuity is normal. A. bacterial conjunctivitis B. viral conjunctivitis C. allergic conjunctivitis D. autoimmune conjunctivitis

A. Question 5 Explanation: This patient has a primary bacterial conjunctivitis. Unlike viral conjunctivitis, bacterial conjunctivitis will produce a mucopurulent discharge from the beginning. Symptoms are more often unilateral, and associated eye discomfort is common. In bacterial conjunctivitis, normal visual acuity is always maintained. There is usually uniform engorgement of all the conjunctival blood vessels. There is no staining of the cornea with fluorescein. Bacterial conjunctivitis should be treated with antibiotic drops such as sodium sulfacetamide, gentamicin, or fluoroquinolones.

A patient presents complaining of left eye discharge and eyes that were matted shut this morning. The patient denies changes in visual acuity, but states that he is afraid to put his contacts in. On physical examination you note erythematous conjunctivae and mucopurulent discharge of the left eye. The cornea is clear. Which of the following topical agents is the treatment of choice in this patient? A. Aminoglycoside (Tobrex) (Fluoroquinolone) B. Olopatadine (Patanol) C. Cycloplegic (Steroid) D. Prednisolone acetate

A. Question 9 Explanation: Topical aminoglycoside or fluoroquinolones are indicated in contact lens wearers with conjunctivitis to cover for Pseudomonas infection.

Retinal detachment occurs when the inner layers of the retina separate from which of the following structures? A Choroid B Fovea centralis C Optic nerve D Sclera

A. The retina is made up of multiple layers of neurons that form a lining in the posterior aspect of the eye. The retina works to convert photons into impulses that the optic nerve then carries to the visual cortex of the brain. The receptors in the retina are among the most metabolically active cells in the entire body and are located in the outer part of the retina. This outer portion is in direct contact with both the underlying choroid and the retinal pigment epithelia (RPE). Retinal detachment occurs when the retina detaches from the choroid and RPE. Patients present with complaints of seeing flashes of light, the presence of floaters, or with a visual field defect. Initial evaluation is with visual acuity testing, evaluating visual fields, dilated retinal exam, and slit lamp biomicroscopy. Treatment is with surgery. The fovea centralis (B) is a shallow depression located at the posterior of the retina. It is the point of greatest visual acuity. The optic nerve (C) carries signals initiated by the presence of photons after electrical and chemical reactions convert them to electrical signals that the optic nerve can relay to the visual cortex of the brain. The sclera (D) is the outermost layer of the eye and is made up of fibrous tissue. Question: What is the most common type of retinal detachment? Answer: Rhegmatogenous retinal detachments. Retinal Detachment Patient will be complaining of painless loss of vision, floaters, flashing lights, curtain lowering sensation PE will show retina appears hazy gray with white folds Management is stat ophthalmology consult

What is the most common cause of permanent legal blindnes and visual loss in elderly patients 75 years and older? A. Macular degeneration B. Diabetic retinopathy C. Chronic open angle glaucoma D. Acute angle closure glaucoma

A. Macular degeneration

During a round-table discussion with your ophthalmology team, you are asked to describe the mechanism of action of certain glaucoma medications. Which of the following correctly matches the drug to its action? A Brimonidine : carbonic anhydrase inhibitor B Dorzolamide : prostaglandin analog C Lantanoprost : selective alpha-2-agonist D Timolol : nonselective beta adrenergic antagonist

Anyone else get PTSD from physio then with this question...no?...just me?....okay... Correct Answer ( D ) Explanation: There are several classes of medications used in the treatment of open-angle (chronic) glaucoma. Once diagnosed, most glaucoma can be successfully managed to prevent the onset or progression of vision loss. The nonselective beta adrenergic antagonists decrease aqueous humour production. Common medications in this group are timolol and betaxolol. Brimonidine (A) is a selective alpha-2-agonist that decreases aqueous humour production and increases its drainage through the trabecular meshwork. Dorzolamide (B) is a carbonic anhydrase inhibitor that decreases production of aqueous humour. Lantanoprost (C) is a prostaglandin analog that increases drainage of aqueous humour. Question: Pilocarpine is another glaucoma medication. What is its mechanism of action? Answer: Cholinergic (causes miosis, increased aqueous humour drainage). Acute Angle-Closure Glaucoma Patient will be entering a dark room or movie theater Complaining of acute unilateral painful vision loss, vomiting, and seeing halos around lights PE will show cloudy cornea and fixed mid-dilated pupil Testing will show IOP ( > 21 mmHg) Treatment is topical beta-blockers, carbonic anhydrase inhibitors, mioticsIridotomy is definitive treatment, but not used for initial management

A 57 year-old male was working on his farm, when some manure was slung hitting his left eye. He presents several days after with a red, tearing, painful eye. Fluorescein stain reveals uptake over the cornea looking like a shallow crater. Which of the following interventions would be harmful? A. Ophthalmic antibiotics B. Pressure patch C. Examination for visual acuity D. Copious irrigation

B. Question 2 Explanation: Patching of the eye after abrasion associated with organic material contamination is contraindicated due to increased risk of fungal infection.

The most common organism isolated in periorbital cellulitis in vaccinated children in the absence of trauma is A. H. influenzae type B B. Streptococcus pneumoniae C. Moraxella catarrhalis D. Staphylococcus aureus E. Pseudomonas aeruginosa

B. Question 2 Explanation: Periorbital and orbital cellulitis may be caused by trauma (e.g., a wound, an insect bite), an associated infection (e.g., sinusitis), or seeding from bacteremia. Before widespread immunization, Haemophilus influenzae type B was the most common cause secondary to bacteremia (about 80% of cases) and remains so in nonimmunized populations. Streptococcus pneumoniae accounted for most of the remaining 20% of cases. S. pneumoniae is the most likely agent in Haemophilus influenzae type B-vaccinated patients when sinusitis is present. The most common pathogens associated with external foci (trauma) are Staphylococcus aureus and Streptococcus pyogenes, but these are seldom isolated from the blood. In general, a bacterial pathogen is isolated from the blood in < 33% of patients with periorbital cellulitis. CT W/ contrast to evaluate abscess formation and extent of disease - Admit, Start Vanco to cover MRSA

A 55 year old female presents with acute painless visual loss in the right eye that occurred just prior to arrival. The patient reports that she was watching television and her right eye "went black". She denies any headache, recent trauma, dizziness, palpitations or photophobia. Her past medical history is significant for CAD, HTN, HLD. On exam, the patient has no perception to light or hand motion in the right eye. The left eye is 20/20. A dilated fundoscopic exam is performed using a slit lamp and the following is seen (Flip card to see picture DONT CHEAT). What are the two distinguishing findings from the image that are pathogonomic for the appropriate diagnosis? A. Cherry retina and pale macula B. Cherry red macula and pale retina C. Retinal hemorrhages with "blood and thunder" and pale retina D. Cupping of the optic nerve and cherry red retina

B. Cherry red macula and pale retina Correct answer: (B) Cherry red macula and pale retina. Explanation: The above fundoscopic exam findings and history are consistent with Central Retinal Artery Occlusion (CRAO). CRAO is thrombus or embolus of the central retinal artery most commonly caused by artherosclerotic disease in patients 50-80 years old. CRAO presents with acute, sudden onset of painless monocular vision loss. Diagnosis is based off history and fundoscopy which reveals a pale retina with a cherry -red macula (red spot) which occurs due to obstruction of retinal blood flow. Veins may also be segmented having a "box car appearance". Emboli may be visualized in 20% of cases and is absent of hemorrhage. Treatment; this condition is an ophthalmological emergency and immediate consult should be obtained. No treatments have been shown effective, however decreasing the intraocular pressure to prevent anterior chamber involvement should occur using acetazolamide. Other treatments include laying the patient flat on their back and massaging the orbit in an attempt to dislodge the clot. Vessel dilation may also be performed as well.

A patient presents with monocular pain, photophobia, tearing and decreased visual acuity. A branching lesion detected with fluorescein staining is pictured below. What is the most likely diagnosis? A. Bacterial conjunctivitis B. Herpes Simplex virus (HSV) keratitis C. Anterior uveitis D. Optic neuritis

B. Herpes Simplex Virus (HSV) Keratitis

A 39-year-old female lifeguard presents with a slow-growing lesion on her eye that has increased in size over the past few years. The patient has no visual changes or discomfort. What is the appropriate management at this time? A. Surgical resection B. Observation C. Scleral buckle D. Topical antifungal drops

B. Observation Patient has a pterygium which no vision changes or discomfort.

A pterygium can be distinguishe dfrom a pinguecula based on history alone. What is the most distinguishing factor between these conditions? A. Pinguecula enlarges slowly, but pterygium size is constant. B. Pinguecula does not enlarge over time, but pterygium does C. Both conditions grow rapidly. D. Pinguecula grows rapidly, but pterygium size is constant

B. Pinguecula does not enlarge over time, but pterygium does

Which of the following causes acute painful loss of vision? A Central retinal artery occlusion B Central retinal vein occlusion C Optic neuritis D Retinal detachment

C. Optic neuritis, an inflammatory, demyelinating disease of the optic nerve, presents as an acute monocular loss of vision. It is more frequently seen in young women. Although often idiopathic, approximately 30% of patients will develop multiple sclerosis (MS) within five years. Vision loss, most commonly a loss of central vision, and loss of color perception, develop over a period of hours and peaks in about one week. Eye pain, typically worse with eye movements, occurs in about 90% of patients. On examination, an afferent pupillary defect is usually present as is a swollen disk. Diagnosis is made based on history and physical exam findings. Magnetic resonance imaging of the brain and orbits with gadolinium will confirm the diagnosis and is helpful in determining those patients at risk for developing multiple sclerosis. After peak vision loss at one to two weeks, symptoms will gradually improve. Intravenous corticosteroids have been shown to increase the rate of recovery, as well as delay the onset of multiple sclerosis, although long-term visual outcome and rate of developing MS at five years are not affected by this treatment. Central retinal artery occlusion (A) presents with acute, painless loss of vision. On examination, the retina is edematous and the fovea is seen as a cherry-red spot. Central retinal vein occlusion (B) also presents with acute, painless loss of vision. Funduscopic examination reveals hemorrhage described as "blood and thunder". A retinal detachment (D) is typically painless. Patients describe flashing lights and floaters, with vision loss curtain-like in appearance. Optic Neuritis Patient will be complaining of acute monocular vision loss, pain worse with eye movements, loss of color (red) vision, and transient worsening of vision with increased body temperature (Uhthoff's phenomenon) Diagnosis is made clinically. MRI will confirm demyelination Most commonly caused by multiple sclerosis Treatment is methylprednisolone IV

A 16 year-old male involved in a fight sustained a laceration to his right upper eyelid. He is unable to open his eye, and a possible laceration of the globe is suspected. Which of the following is the next step? A. Use a slit lamp to determine the extent of the injury. B. Use fluorescein strips to determine the extent of injury. C. Apply a metal eye shield and refer to an ophthalmologist. D. Apply antibiotic ointment to the lid and recheck in 24 hours.

C. Question 3 Explanation: Protect the eye from any pressure with a rigid metal eye shield and refer for immediate ophthalmologic consultation. Avoid unnecessary actions that would delay treatment or cause further injury.

A 64 year-old woman complains of headache and left eye pain for about a day. She says it started yesterday as a dull ache and now is throbbing. She also complains of nausea and vomiting, which she attributes to the popcorn she ate at the movie theater yesterday afternoon. On exam, the left pupil is mid-dilated and nonreactive. The cornea is hazy. A ciliary flush is noted. Which of the following is the most likely diagnosis? A. Migraine headache B. Temporal arteritis C. Acute glaucoma D. Retinal artery occlusion

C. Question 1 Explanation: Acute glaucoma often presents with abdominal complaints that may delay diagnosis. Findings of ciliary flush, mid-dilated and nonreactive pupil, and hazy cornea in a patient with severe eye pain are consistent with acute angle closure glaucoma.

A 29-year-old man presents to the emergency department with bilateral eye pain. The patient states it has slowly been worsening over the past 48 hours. He admits to going out this past weekend and drinking large amounts of alcohol and having unprotected sex but cannot recall a predisposing event. The patient's vitals are within normal limits. Physical exam is notable for bilateral painful and red eyes with opacification and ulceration of each cornea. The patient's contact lenses are removed and a slit lamp exam is performed and shows bilateral corneal ulceration. Which of the following is the best treatment for this patient? A. Acyclovir B. Erythromycin ointment C. Gatifloxacin eye drops D. Intravitreal vancomycin and ceftazidime E. Topical dexamethasone and refrain from wearing contacts

C. Question 1 Explanation: This patient is presenting with painful, red eyes with opacification and ulceration of each cornea in the setting of contact lens use suggesting a diagnosis of contact lens-associated infectious keratitis. Treatment for contact lens-associated infectious keratitis includes topical broad-spectrum antibiotics (such as gatifloxacin eye drops). Bacterial possibilities = Staph A. Pseudomonas, Coag-Negative Staph, Diphtheroids, Strep Pneumo

A patient with type 2 diabetes mellitus presents for a yearly eye exam. Ophthalmoscopic exam reveals neovascularization. Which of the following is the most likely complication related to this finding? A. Glaucoma B. Cataracts C. Vitreous hemorrhage D. Optic Neuritis

C. Question 2 Explanation: Proliferative retinopathy, as evidenced by neovascularization, is associated with an increased risk of vitreous hemorrhage.

A 7 year old boy presents to your clinic with thick purulent discharge from her right eye which was significantly worse this morning. What should you do next? A. take a sample of the discharge and send it to the lab for culture. B. send the patient home to perform warm compresses for 20 minutes 3 times daily tell them to come back if symptoms become worse C. moxifloxacin 0.5% drops tid for 7 to 10 days D. topical 1% prednisolone acetate qid

C. Question 3 Explanation: If neither gonococcal nor chlamydial infection is suspected, most clinicians treat presumptively with moxifloxacin 0.5% drops tid for 7 to 10 days or another fluoroquinolone or trimethoprim/polymyxin B qid. A poor clinical response after 2 or 3 days indicates that the cause is resistant bacteria, a virus, or an allergy. Culture and sensitivity studies should then be done (if not done previously); results direct subsequent treatment.

A 79-year-old woman presents with a dry, painful right eye. During inspection, you notice her lower lid has turned outward on itself, pulling away from the eyeball. Slit lamp examination shows no abnormalities. The conjunctiva do not appear injected, and the iris has no erythema. Which of the following is the most likely diagnosis? A Anterior uveitis B Corneal abrasion C Ectropion D Keratitis

C. Ectropion, the outward turning of the lower eyelid, is mainly due to the aging process. Weakening of the periorbital connective tissue allows the eyelid to fall away from the eyeball, leading to dry painful eyes. Associated symptoms include excessive tearing, blepharitis and non-infectious conjunctivitis. Other causes include scarring from burn injuries, congenital disease such as Down syndrome, and cranial nerve 7 palsies. Artificial tears may be of help, but if not, then treatment is surgical. Left untreated, complications of infection, corneal abrasion or corneal ulcer may arise. As a general rule, any red eye with pain should prompt an ophthalmology consultation. Anterior uveitis (A), also known as iritis or iridocyclitis, is inflammation of the iris (pigmented fibrovascular tissue and epithelium) and anterior chamber (posterior to cornea, contains aqueous humour). Corneal abrasion (B) refers to a defect in the corneal epithelium, typically from some mechanical injury. Slight lamp examination with fluorescein stain would typically illuminate an abrasion of the cornea. Keratitis (D) is the term used for generalized corneal inflammation, which is not present in the above patient. Ectropion Patient will be complaining of tearing and symptoms of dry eyes PE will show red conjunctiva and the eyelid turns outward Most commonly caused by aging

On fundoscopic exam of a patient with sudden acute painless monocular visual loss, you observe extensive retinal hemorrhages that have a "blood and thunder appearance". Which of the following is a known risk factor of central retinal vein occlusion? A. Age > 50 B. Genetic predisposition C. Hypertension D. Cigarette smoking

C. HTN

A 26-year-old male presents to the emergency room after a physical altercation. The patient reports being struck multiple times in the face. He denies a loss of consciousness but complains of severe left-sided periorbital pain and monocular diplopia when he attempts to look upward. The patient also complains of blurred vision in the left eye and numbness of the left cheek bone. On physical exam, the left eye ball is posteriorly displaced. There is subcutaneous emphysema of the left zygoma as well as slight mobility. Sensation of the anteromedial aspect of the zygoma is significntly decreased. Extraocular movement of the left eye is impaired. X-ray of the facial bones deonstrates a "tear-drop sign". Based on the X-ray and physical exam findings, damage to what nerve is causing this patient's zygoma paresthesia? A. Supraorbital B. Supratrochlear C. Infraorbital D. Zygomaticotemporal

C. Infraorbital

A 2-year old child presents with right eye swelling. CT shows inflammation of the eyelid, but not the orbit. What is the recommended treatment? A. Ocular decompression alone B. Ednoscopic sinus surgery and IV abx C. PO abx w/ close FUP D. IV abx and heparin infusion E. IV abx, heparin infusion and endoscopic sinus surgery

C. PO abx w/ close FUP

A 32-year-old man presents with a "lump" in his eyelid for the past several weeks. He denies any pain. Physical exam shows a hard, non-tender nodule on the lateral aspect of the right upper eyelid. Which of the following findings in this patient's history would be most consistent with the diagnosis? A Anterior blepharitis B Diabetes mellitus C Hyperlipidemia D Working outdoors frequently

Correct Answer ( A ) Explanation: A history of anterior blepharitis increases the potential risk of developing a chalazion. A chalazion is a granulomatous, inflammatory condition of a Meibomian gland that typically presents as a non-tender mass of the eyelid. The condition sometimes follows the development of an internal hordeolum that does not drain. Treatment is with warm compresses. If the chalazion becomes inflamed or impacts vision, then removal may be necessary with incision and drainage. Another less invasive therapy is glucocorticoid injection. Diabetes mellitus (B) does not increase the risk of chalazion. Hyperlipidemia (C) increases the risk of developing xanthelasma which are cholesterol-filled, soft plaques. Working outdoors frequently (D) may expose the patient to more sand and sun exposure. These increase the risk of developing pterygium and pinguecula. Question: Describe a serious complication of a chalazion. Answer: Large chalazia can press on the cornea causing astigmatism. Chalazion Patient will be complaining of a painless mass on the upper eyelid Most commonly caused by obstruction of Meibomian gland Treatment is warm compresses Topical antibiotics if progression to blepharitis/cellulitis.

A teenage girl presents with an eyelid lump that began 3 months ago. It has noticeably enlarged over the past 2 weeks. On inspection, you appreciate a mass which seems to be originating on the internal surface of the superior right eyelid. Palpation of this mass reveals a non-greasy, non-tender, non-mobile nodule. Attempted expression of this nodule offers no discharge. Which of the following is the most likely diagnosis? A Chalazion B Hordeolum C Nevus D Seborrheic keratosis

Correct Answer ( A ) Explanation: A slow growing, chronic subcutaneous eyelid nodule is called a chalazion. These nodules are typically non-tender, and can occur within the lid, or on the internal or external surface of the lid. Obstruction of the meibomian gland is the most common cause, leading to formation of a lipogranulomatous mass, which is usually non-tender. Chalazia may or may not become secondarily infected, and they may or may not be accompanied by blepharitis (generalized eyelid inflammation). Treatment consists of lid hygiene, topical antibiotics and ultimately, intralesional steroids. A hordeolum (B), or stye, is a more acutely presenting, painful eyelid lesion. It more commonly occurs on the lid margin, but can also occur on the internal surface. Internal surface eyelid lesions typically develop from plugging of a meibomian gland, while lid-margin or external surface eyelid lesions may represent infection of the more anteriorly located glands of Moll/Zeiss. Hordeola usually self-drain within one week of lid hygiene and antibiotic ointment treatment. Nevi (C) are well-demarcated, usually flat or minimally elevated, pigmented or nonpigmented lesions that are present at birth. This patient's lesion began 3 months ago. Seborrheic keratosis (D) is a pigmented, greasy, hyperkeratotic lesion that appears to be stuck on the skin. They are more common in the elderly population. Question: Which cosmetic side effect may result from steroid injection of a chalazion? Answer: Hypopigmentation. Chalazion Patient will be complaining of a painless mass on the upper eyelid Most commonly caused by obstruction of Meibomian gland Treatment is warm compresses Topical antibiotics if progression to blepharitis/cellulitis.

A 57 year-old man presents to the office with a chief complaint left eye pain and seeing halos around lights. His left eye is seen above. Which of the following is the most likely diagnosis? (Cornea is hazy, mid-dilated pupil and perilimbic injection) A Acute angle closure glaucoma B Acute anterior uveitis C Bacterial conjunctivitis D Hyphema

Correct Answer ( A ) Explanation: Acute angle-closure glaucoma is an ophthalmologic emergency due to a closed anterior chamber angle. This is the only type of glaucoma that is curable. Patients present with acute onset of pain and vision loss that is most commonly unilateral. As intraocular pressure rises, patients may experience headache, nausea, vomiting, and abdominal pain that is sometimes mistaken as an acute abdominal process. The eye exam may reveal a red, tender globe that may be firm to touch. The cornea may be steamy or hazy and the pupil is mid-dilated and nonreactive. An ophthalmologic consult should be obtained immediately. Treatment incudes topical drops such as carbonic anhydrase inhibitors and beta-blockers. Permanent vision loss can occur if treatment is delayed. Acute anterior uveitis (B) presents with a sudden onset of unilateral eye redness, photophobia, pain and lacrimation. It is associated with systemic illnesses including anklosing spondylitis, Crohn's, ulcerative colitis, reactive arthritis and sarcoidosis. Bacterial conjunctivitis (C) is a self-limiting condition that is characterized by acute onset of redness in one or both eyes, a gritty or burning sensation and discharge that begins watery and eventually becomes purulent. Many patients awaken with their eyelids stuck together. Treatment consists of antibiotic eye drops or ointments. A hyphema (D) is usually secondary to globe trauma and is caused by hemorrhage from either the ciliary body or the anterior chamber. Question: What condition are children with H. Influenzae conjunctivitis prone to develop? Answer: Otitis Media. Acute Angle-Closure Glaucoma Patient will be entering a dark room or movie theater Complaining of acute unilateral painful vision loss, vomiting, and seeing halos around lights PE will show cloudy cornea and fixed mid-dilated pupil Testing will show IOP ( > 21 mmHg) Treatment is topical beta-blockers, carbonic anhydrase inhibitors, mioticsIridotomy is definitive treatment, but not used for initial management

A 52-year-old man presents with bilateral eye irritation and itching for the last one week. Physical exam reveals a "red-rimmed" appearance to the eyes with scaling noted at the lashes. Which of the following is the most likely diagnosis? A Anterior blepharitis B Dacryocystitis C Ectropion D Internal hordeolum

Correct Answer ( A ) Explanation: Anterior blepharitis is a common inflammatory condition of the eyes involving the lid margins. In anterior blepharitis, the eyelashes are typically involved and physical exam may demonstrate scaling or granulations at the lid margins. The eyes may also appear "red-rimmed". The condition can be associated with other areas of seborrheic dermatitis or rosacea. Treatment involves lid hygiene, including washing the lids daily with a gentle shampoo (such as baby shampoo) that is not irritating to the eyes. Acute exacerbations and inflammation can be treated with either topical bacitracin or erythromycin ointment applied to the lid margins. In severe exacerbations, topical prednisolone or oral antibiotics may be necessary. Dacryocystitis (B) is an infection of the lacrimal sac and does not present with bilateral lid swelling and erythema. Ectropion (C) is an eversion of the lower eyelid that is common in older age. An internal hordeolum (D), commonly referred to as a stye, presents as a painful swelling that is typically unilateral in the inner lid margin. Question: What is the most common complication of anterior blepharitis? Answer: Recurrent conjunctivitis. Blepharitis Patient will be complaining of eyelid changes and eyelash flaking PE will show crusting, scaling, and red-rimming of the eyelid Diagnosis is made by slit-lamp examination Most commonly caused by dysfunctional meibomian gland Treatment is warm compresses, irrigation, lid massage, and topical antibiotics for flare-ups Comments: associated with seborrhea and rosacea

A 52-year-old man presents with a red right eye that is associated with severe pain and blurred vision. On exam there is some cloudiness to the cornea. Pupils are 5 mm OD and 3 mm OS. Which of the following is the most likely diagnosis? A Acute glaucoma B Conjunctivitis C Iritis D Keratitis

Correct Answer ( A ) Explanation: Glaucoma is characterized by an increase in intraocular pressure leading to irreversible damage to the optic nerve. There are two forms of glaucoma: open-angle and angle-closure. Open-angle glaucoma is the most common form of glaucoma and is most often seen in the elderly population. Acute angle-closure glaucoma is an ophthalmologic emergency and is caused by closure of the anterior chamber. Symptoms are rapid onset and usually unilateral. It presents with severe pain and profound vision loss. As intraocular pressure increases, patients may experience headache, nausea, and vomiting. Exam reveals a red, tender globe that may be firm to touch. A steamy or hazy cornea may be seen with a nonreactive, dilated pupil. Treatment should be initiated to reduce the intraocular pressure. Patients require emergent ophthalmologic consultation. Conjunctivitis (B) is not associated with severe pain or a dilated pupil. Iritis (C) is typically associated with ciliary flush, blurred vision, and a clear cornea. The pupil in keratitis (D) is reactive and not dilated. Question: What is the quickest way to diagnose acute angle glaucoma? Answer: Elevated intraocular pressure using tonometry. Acute Angle-Closure Glaucoma Patient will be entering a dark room or movie theater Complaining of acute unilateral painful vision loss, vomiting, and seeing halos around lights PE will show cloudy cornea and fixed mid-dilated pupil Testing will show IOP ( > 21 mmHg) Treatment is topical beta-blockers, carbonic anhydrase inhibitors, mioticsIridotomy is definitive treatment, but not used for initial management

What is the most common infectious etiology of conjunctivitis in adults? A Adenovirus B Enterovirus C Staphylococcus aureus D Streptococcus pneumoniae

Correct Answer ( A ) Explanation: Viral conjunctivitis is the most common type of conjunctivitis in adults and is most often caused by adenovirus. Patients present with symptoms of redness, watery or mucoserous discharge, and irritation. It is typically bilateral (but may initially affect only one eye) and there is often a preceding or concomitant upper respiratory illness. Physical examination findings include preauricular lymphadenopathy, diffuse conjunctival injection, and enlarged follicles on the inferior palpebral conjunctiva. Visual acuity, examination of the cornea and anterior chamber, and intraocular pressures are normal. Treatment consists of supportive care with cool compresses. Some patients may get relief from an over-the-counter topical antihistamine or ocular decongestant but this treats only the symptoms and not the cause. Topical antibiotics are reserved for those patients in whom the etiology is unclear or if there is concern for a superimposed bacterial infection. Enterovirus (B) may also cause viral conjunctivitis, but is less frequently implicated than adenovirus. Staphylococcus aureus (C) and Streptococcus pneumoniae (D) are common causes of bacterial conjunctivitis, but neither are responsible for the majority of cases overall. Question: What findings are seen in keratoconjunctivitis? Answer: Keratoconjunctivitis is infection of the cornea in addition to the conjunctiva. Findings include eye pain and redness, photophobia, conjunctival injection and diffuse, fine, fluorescein uptake on the cornea. Viral Conjunctivitis Patient will be complaining of bilateral red/pink eye PE will show preauricular lymphadenopathy, copious watery discharge, scant mucoid discharge Most commonly caused by adenovirus Treatment is cool compresses, artificial tears, antihistamines for itching/redness

A 38-year-old man presents to the ED with his friend. They were playing racquetball and the man was hit directly in the eye with the ball. Examination reveals limitation of upward gaze, periorbital erythema, and subconjunctival hemorrhage of right eye. Imaging of the orbit would most likely show a fracture of which bone? A Frontal B Maxillary C Sphenoid D Zygomatic

Correct Answer ( B ) Explanation: A blowout fracture typically refers to a fracture of the inferior orbital wall fracture. The most common causes are direct blunt injury such as a fist, sports' ball injury, and motor vehicle collisions. Symptoms include upward gaze diplopia, decreased extraoccular eye movements due to entrapment of the inferior rectus muscle, periorbital ecchymoses, eyelid edema and subconjunctival hemorrhage. Suspicion is based on the history, while diagnosis requires radiographic or computed tomography confirmation. Typical radiographic findings include a teardrop sign (a mass of herniated orbital contents, fat and the inferior rectus muscle), orbital lucency, and sinus air-fluid levels. Pure blowout fractures occur most frequently in the inferior orbital floor. Since the floor (inferior wall) is the most common site of orbital blowout, the maxillary bone is most commonly fractured. The medial wall is the second most commonly fractured area. The orbit is a pyramidal-like structure made up of 6 areas: superior orbital wall (frontal bone) (A), inferior orbital wall or floor (maxillary bone), medial wall (lacrimal and ethmoid bones), lateral wall (zygomatic bone) (D), apex (sphenoid bone) (C), and base (anterior eyeball surface). Question: When is surgery recommended for treating an orbital blow-out fracture? Answer: Fracture >50% of the floor, extraoccular muscle entrapment, enophthalmos or diplopia. Orbital Blowout Fracture Patient with a history of direct trauma to the orbit Complaining of limitation of upward gaze, infraorbital anesthesia PE will show inferior rectus entrapment, enophthalmos CT will show teardrop sign Diagnosis is made by CT Management includes ABX, decongestants

A 32-year-old man complains of left eye pain and foreign body sensation. He reports associated tearing and photophobia. He was grinding metal without wearing protective eye gear. An eye exam with fluorescein is performed as shown above. What is the most likely diagnosis? A Acute-angle closure glaucoma B Corneal abrasion C Corneal ulceration D Traumatic iritis

Correct Answer ( B ) Explanation: Corneal abrasions are usually detected using fluorescein stain under a Wood's lamp or cobalt blue light examination. This can also help detect a corneal foreign body. Typically, corneal abrasions are caused by eye trauma. Symptoms include pain, conjunctival injection, photophobia, and a foreign body sensation. If a penetrating injury is suspected, spiral CT may be indicated to detect a ruptured globe or a nonmetallic foreign body. Patients should be prescribed topical antibiotics and ophthalmic nonsteroidal anti-inflammatory drugs can be considered for pain. Prognosis is generally favorable with most corneal abrasions healing within two to three days after injury. If the pain, redness, or photophobia persists then the patient should be referred to ophthalmology. Acute-angle closure glaucoma (A) manifests as a cloudy cornea and scleral injection and is usually accompanied with severe pain and vision loss. Corneal ulceration (C) is typically associated with diffuse erythema with ciliary injection, eye discharge, and may involve hypopyon and vision loss. Traumatic iritis (D) causes severe photophobia (direct and consensual), perilimbal injection, and vision loss. Question: What does a positive Seidel's sign indicate? Answer: Globe perforation. Corneal Abrasion Patient will be complaining of pain, photophobia, tearing, or foreign body sensation Diagnosis is made by fluorescein stain Treatment: Most are self-limiting,anti pseudomonal for contact lens wearers

A 34-year old woman is diagnosed with rheumatoid arthritis and started on hydroxychloroquine. Which of the following should be monitored while she is taking this medication? A ECG B Eye exam C Liver transaminases D Serum creatinine

Correct Answer ( B ) Explanation: Hydroxychloroquine is an antimalarial agent that is used in the treatment of rheumatoid arthritis due to its minimal side effects and low toxicity. However, use of hydroxychloroquine, as monotherapy, is limited to only mild, early cases due to a three- to six-month time frame of response. Antimalarial agents alter antigen degeneration by elevating cellular pH, thus the presentation of antigen to T-cells is diminished. Hydroxychloroquine can be combined with other disease-modifying antirheumatic drugs (DMARDS), such as methotrexate or sulfasalazine. With long-term use, hydroxychloroquine can cause irreversible damage to the retina. Although the mechanism behind this toxicity has not been established, it is thought that hydroxychloroquine alters retinal cell metabolism and binds to melanin. Risks for toxicity include daily doses greater than 400 mg, medication use for more than five years, concomitant liver or renal disease, underlying retinal disease, and age greater than 60 years. The American Academy of Ophthalmology recommends a baseline ophthalmologic examination before beginning hydroxychloroquine and then annually while the patient is on the medication. A CBC should be performed periodically if anemia of chronic disease due to rheumatoid arthritis is present to monitor for improvements. Additionally, hydroxychloroquine adverse effects include agranulocytosis, thrombocytopenia, and aplastic anemia. Hydroxychloroquine is not known to cause any conduction abnormalities, so an ECG (A) is not needed for routine monitoring. Liver transaminases (C) and serum creatinine (D) should be drawn prior to initiating treatment since a decrease in kidney and liver function may result in increased retinal toxicity. However, unless signs of renal or hepatic dysfunction arise, routine monitoring is not required. Question: Hydroxychloroquine use should be avoided in patients with what X-linked recessive disorder? Answer: Glucose-6-phosphate dehydrogenase deficiency. Hydroxychloroquine (Plaquenil) DMARD Anti-malarial drug Used for RA, Lupus, JIA Yearly eye exam

A 33-year-old man presents to the emergency department with a left eye injury that occurred just prior to arrival. Patient states that he was working with a table saw and a piece of wood flew into his left eye. He has decreased visual acuity, eye pain, profuse tearing, photophobia, and difficulty opening his eye. He has a foreign body sensation as well. Which of the following findings on physical exam would suggest globe perforation? A A negative Seidel test B Conjunctival injection C Hyphema in the anterior chamber D Increase intraocular eye pressure

Correct Answer ( C ) Explanation: A hyphema of the anterior chamber is suggestive of globe perforation or penetration. Patients presenting with a history of trauma to the eye, particularly with a foreign body, should always be examined for signs of globe and attrition. Examination with fluorescein and a UV lamp will demonstrate a foreign body if one is present. Corneal foreign bodies are usually superficial and most are easily removed under local anesthesia. Foreign bodies may be irrigated off of the cornea with normal saline or dislodged with a moistened cotton applicator. If a foreign body is embedded in the cornea a small gauged needle can be used to remove the foreign body under UV lamp examination. Knowing the material makeup of the corneal foreign body is important. Metallic foreign bodies can leave rust rings behind in the cornea that can be toxic to tissue. These patients will need to be followed by an ophthalmologist and an ophthalmic burr or spud can be used to remove the rust ring at the follow-up visit within 24 hours. Topical antibiotics should be prescribed and tetanus status should be updated. A negative Seidel test (A) does not indicate globe perforation. A positive Seidel test may be apparent with penetration of the globe. Conjunctival injection (B) is nonspecific and does not suggest globe penetration. Intraocular eye pressure (D) should not be measured if there is a suspicion of a globe penetration. If intraocular eye pressure is measured, it may be decreased. Question: What is the low-speed, low torque, hand-held drill called that can be used to remove a corneal foreign body called? Answer: An alger brush. Hyphema Patient with a history of blunt or penetrating trauma Complaining of blurry vision PE will show unequal pupils, injected conjunctiva/sclera, and blood in anterior chamber Treatment is with eye protection, limitation of activity, and head elevation of 30-45 degrees

A 55-year-old mechanic complains of irritation in his right eye lasting for 2 days. On direct visualization you see a small, dark foreign body on the periphery of the cornea and are unable to remove it. There is a patch of reddish-brown discoloration extending several millimeters around the area where the foreign body is located. Which one of the following is most appropriate for this patient? A An antibiotic ointment to be used every 2-4 hours B Irrigation with 0.9% saline solution under pressure C Prompt ophthalmologic evaluation D Watchful waiting

Correct Answer ( C ) Explanation: Corneal foreign bodies generally fall under the category of minor ocular trauma. Small particles may become lodged in the corneal epithelium or stroma, particularly when projected toward the eye with considerable force. The foreign object may set off an inflammatory cascade, resulting in dilation of the surrounding vessels and subsequent edema of the lids, conjunctiva, and cornea. White blood cells also may be liberated, resulting in an anterior chamber reaction or corneal infiltration. If not removed, a foreign body can cause infection or tissue necrosis. If a metal foreign body is present on the cornea for more than 24 hours a rust ring will often be present in the superficial layer of the cornea. This material is toxic to the cornea and should be removed as soon as possible. Rust rings that remain in the cornea after removal of a metallic foreign body may require removal with a slit lamp and rust ring drill. Referral to an ophthalmologist within 24-48 hours is the best management in this case. Antibiotics (A) may be prescribed to prevent infection of an open corneal abrasion, however removal of the foreign body is the crucial step to treatment. Irrigation with 0.9% saline solution (B) under pressure and watchful waiting (D) are not sufficient when a toxic material such as metal is introduced to the eye. Question: What type of metal is responsible for forming rust rings? Answer: Iron. Corneal Abrasion Patient will be complaining of pain, photophobia, tearing, or foreign body sensation Diagnosis is made by fluorescein stain Treatment: Most are self-limiting,anti pseudomonal for contact lens wearers

Which of the following ophthalmologic devices aids in the diagnosis of glaucoma? A Phoropter B Slit lamp C Tonometer D Wood's lamp

Correct Answer ( C ) Explanation: Glaucoma, the leading cause of blindness in the world and the second most common cause of blindness in the US, refers to a secondary optic neuropathy. It may or may not be due to increased intraocular pressure. Increased intraocular pressure is largely due to poor aqueous humour flow between the anterior and posterior eye chambers through the trabecular meshwork. There are four main types. Congenital glaucoma is rare. Secondary glaucoma occurs after trauma or injury. Closed-angle glaucoma, an ophthalmologic emergency, usually presents as an acute loss of vision, feeling of eye swelling, cloudy or steamy vision with halos around lights and eye pain. Open-angle glaucoma, the most common type, is a chronic, slowly progressive disease of painless loss of vision, in which most patients have no symptoms until late in the disease. Diagnosis requires tonometry testing, in which the intraocular pressure is measured directly. Visual loss can be prevented if glaucoma is diagnosed early, as such, regular ophthalmological exams are important. Treatment options include eye drops, oral medications, laser procedures or surgery. A phoropter (A) is used during a visual examination to determine refractive error, which is then used to determine an individual's eyeglass prescription ("which is better, 1 or 2, 3 or 4..."). A slit lamp (B) is used with fluorescein stain to detect corneal abrasions, not elevated intraocular pressure. A Wood's lamp (D) uses ultraviolet light to detect corneal abrasions, epithelial bacterial or fungal infections and vitiligo. Question: A yellow-green fluorescence seen during Wood's lamp examination can detect early infection of which bacterium? Answer: Pseudomonas, common in contact lens wearers. Acute Angle-Closure Glaucoma Patient will be entering a dark room or movie theater Complaining of acute unilateral painful vision loss, vomiting, and seeing halos around lights PE will show cloudy cornea and fixed mid-dilated pupil Testing will show IOP ( > 21 mmHg) Treatment is topical beta-blockers, carbonic anhydrase inhibitors, mioticsIridotomy is definitive treatment, but not used for initial management

A 3-year-old boy presents with atraumatic eye pain. Examination reveals a bulging, erythematous, edematous left eye with decreased extraocular movement and surrounding eyelid edema. Which of the following is the most appropriate initial management step? A Consult infectious disease service B Consult ophthalmology C Intravenous antibiotics D Oral antibiotics

Correct Answer ( C ) Explanation: Inflammation behind the orbital septum, the fibrous portions of the eyelids which form the anterior boundary of the orbit, is called orbital cellulitis. Common causes are sinusitis, upper respiratory infections, eye trauma and sepsis. Symptoms include eye pain and limitation with orbital movement, erythema and edema, bulging of the eye and potentially a loss of vision (therefore, it is an ophthalmologic emergency). If untreated, up to 20% of patients will become blind. Inpatient treatment is recommended, and includes hospital admission, labs, cultures, intravenous antibiotics and frequent monitoring with neurologic and vision checks. If improvement is seen in 48 hours, discharge home on oral antibiotics. If no improvement in 48 hours, consider the presence of an abscess and consult surgery and an infectious disease specialist. Ophthalmology (B) and infectious disease (A) consultation should be obtained after initiation of parenteral antibiotics. Outpatient treatment with oral antibiotics (D) is not recommended. Frequent monitoring is necessary as the condition can lead to blindness. Question: What are the most common organisms that cause orbital cellulitis? Answer: Staphylococcus aureus and Streptococcus pneumoniae. Orbital Cellulitis Patient will be complaining of swelling around the eye PE will show decreased extraocular movement, pain with movement of the eye, and proptosis Diagnosis is made clinically Most commonly caused by S. aureus Treatment is ophthalmology evaluation, antibiotics

A 60-year-old man presents to his primary care provider with complaints of left eyelid swelling and eye pain. Two weeks ago, he was treated at the clinic for sinusitis. Physical exam reveals a febrile man with a swollen and erythematous left eyelid. Extraocular movements are painful. Visual acuity is unaffected. Which of the following is the most appropriate management? A Ciprofloxacin ophthalmic 0.3% ointment B High-dose oral amoxicillin-clavulanate C Intravenous vancomycin and ceftriaxone D Oral cephalexin

Correct Answer ( C ) Explanation: Intravenous vancomycin and ceftriaxone is the most appropriate treatment for orbital cellulitis. Orbital cellulitis is an infection of the ocular muscles and periorbital fat, in contrast to preseptal cellulitis, which only involves the anterior soft tissue. Orbital cellulitis does not involve the globe. Orbital cellulitis is most frequently seen in young children and in older adults. The most common cause of orbital cellulitis is bacterial rhinosinusitis. Staphylococcus aureus is the most frequently identified pathogen in orbital cellulitis. Distinguishing orbital cellulitis from preseptal cellulitis is important because orbital cellulitis is potentially vision and life threatening. Orbital cellulitis and preseptal cellulitis both cause ocular pain and eyelid erythema and swelling however, only orbital cellulitis causes painful eye movements, proptosis, and ophthalmoplegia. Fever and chemosis can occur in severe preseptal cellulitis, but are more common in orbital cellulitis. Computed tomography is used to confirm clinically suspected orbital cellulitis. Blood cultures are recommended in all cases of orbital cellulitis prior to antibiotic initiation. An ophthalmology and otolaryngology consult should obtained for all patients. Patients with orbital cellulitis require empiric intravenous antibiotics aimed at S. aureus. Typical antibiotic regimens include vancomycin plus one of the following: ceftriaxone, cefotaxime, ampicillin-sulbactam, or piperacillin-tazobactam. Ciprofloxacin ophthalmic 0.3% ointment (A), high-dose oral amoxicillin-clavulanate (B), and oral cephalexin (D) are not effective for the treatment of orbital cellulitis, which requires parenteral antibiotics. Ciprofloxacin ophthalmic 0.3% ointment would be appropriate therapy for bacterial conjunctivitis in patients who wear contact lenses. High-dose oral amoxicillin-clavulanate would be appropriate treatment for bite wounds, severe respiratory tract infections, pyelonephritis, and skin infections. Oral cephalexin is commonly used to treat skin infections, streptococcal pharyngitis, uncomplicated cystitis, and otitis media. Question: Which sinus borders the orbit and is the most common route of infection of the orbit? Answer: Ethmoid sinus. Orbital Cellulitis Patient will be complaining of swelling around the eye PE will show decreased extraocular movement, pain with movement of the eye, and proptosis Diagnosis is made clinically Most commonly caused by S. aureus Treatment is ophthalmology evaluation, antibiotics

A 72-year-old man complains of painless, decreased vision in his left eye associated with flashing lights and floaters. Visual acuity in the left eye is 20/200 and in the right eye is 20/30. Which of the following is the most likely diagnosis? A Acute angle-closure glaucoma B Optic neuritis C Retinal detachment D Temporal arteritis

Correct Answer ( C ) Explanation: Retinal detachment involves a separation of the inner neuronal retina from the outer retinal pigment epithelial layer, usually as the result of a tear in the retina from vitreous traction. Early symptoms include flashing lights, the presence of "spider webs" or "coal dust" and floaters in the visual field. As the retina detaches, the patient experiences painless vision loss often described as a curtain that is gradually lowering or raising in front of the affected eye. Direct fundoscopy may reveal a dull, gray detached retina. Management involves immediate ophthalmology consultation for possible surgery or laser therapy. Acute angle-closure glaucoma (A) is due to a rise in intraocular pressure and is associated with painful monocular vision loss. It is associated with a fix mid-dilated pupil, a hazy cornea, and perilimbic injection. Optic neuritis (B) is an inflammatory process of the optic nerve. It is characterized by loss of central vision with preservation of peripheral vision. Patients often describe pain with eye movement and reduced color vision. Temporal arteritis (D) is a vasculitis that causes monocular vision loss and is associated with headache and polymyalgia rheumatica. It is more common in individuals older than 50 years. Question: In retinal detachment with maintained visual acuity, what has happened to the macula? Answer: The macula is likely still attached if visual acuity is maintained. Urgent surgery needed to prevent macula from detaching too. Retinal Detachment Patient will be complaining of painless loss of vision, floaters, flashing lights, curtain lowering sensation PE will show retina appears hazy gray with white folds Management is stat ophthalmology consult

A 52-year-old woman presents with progressive central vision loss. She also complains of seeing "red spots" and "floaters". Her medical history is significant for chronic, poorly controlled hypertension and diabetes. She denies trauma and other neurologic symptoms. Which of the following is most likely to be seen during a fundoscopic examination? A Koplick spots B Lewy bodies C Microaneurysms D Optic disc edema

Correct Answer ( C ) Explanation: Retinopathy can occur as an acute disease, but more commonly, it is a chronic condition. Two of the most common chronic causes in the US are diabetes and hypertension. Other causes include radiation and sunlight exposure, sickle cell disease, retinal vessel occlusion and trauma. Concerning diabetic retinopathy, the underlying pathology is vessel leakage, microaneurysms and hypervascularization. Eventual edema and hemorrhage lead to vision impairment (especially central vision loss), floaters or red spots, and possible blindness. Fundoscopic examination will reveal several findings, the earliest of which is microaneurysm. Dot and blot, flame-shaped and splinter hemorrhages are also common. Cotton-wool spots, tiny infarcts due to arteriolar occlusion, and macular edema are two common findings in later stages of the disease. Treatment includes laser surgery or vitrectomy. Prevention consists of optimal control of serum glucose and cholesterol, as well as blood pressure. Yearly dilated fundoscopic examination is also recommended. Koplick spots (A) are clustered white lesions of the buccal mucosa that occur a few days prior to measles. Lewy bodies (B) are present in brain tissue, not retina, of elders with dementia. Optic disc edema (D) occurs in optic neuritis (papillitis) and papilledema (increased intracranial pressure state). Macular, not optic, disc edema is more common in diabetic retinopathy. Diabetic Retinopathy Patient with a history of diabetes Complaining of progressive central vision loss and of seeing red spots and floaters Fundoscopicexam will show microaneurysms, hypervascularization, flame-shaped and splinter hemorrhages, and cotton-wool spots Treatment is glucose control, laser surgery or vitrectomy DM patients should have yearly eye exam

A 21-year-old woman presents with pain, tearing, photophobia, and left eye redness. She has been wearing her contact lenses continuously for the last two weeks. A slit lamp examination reveals a white, hazy opacity on the cornea at the 3 o'clock position of the cornea with associated limbal flush. Which of the following is the most appropriate pharmacotherapy? A Topical acyclovir B Topical amphotericin B C Topical ciprofloxacin D Topical cyclopentolate

Correct Answer ( C ) Explanation: Topical ciprofloxacin is first-line treatment for a corneal ulceration. This is because of the necessary coverage against Pseudomonas aeruginosa. A corneal ulcer is a bacterial infection that develops secondary to a break in the corneal epithelium. Risk factors for developing a corneal ulcer include incomplete lid closure (e.g. secondary to Bell's palsy) and soft contact lenses use. Symptoms include redness, swelling of the lids, foreign body sensation, and photophobia. Physical exam may reveal a round or an irregular ulceration with a white or hazy base. A secondary iritis can develop and cause ciliary flush or a limbal injection. Treatment includes topical antibiotics and emergent ophthalmologic consultation. Cultures can be obtained of the ocular drainage to guide antibiotic therapy. Cycloplegic drops can be used to help with the pain from the secondary iritis. Complications include corneal scarring, corneal perforation, and secondary glaucoma. Topical acyclovir (A) is used for viral infections, including herpes keratitis. Topical amphotericin B (B) is used to treat fungal infections. Topical cyclopentolate (D) can be used to help with the pain, however, this is not the most appropriate pharmacotherapy as this is a serious bacterial infection that needs to be treated. Question: What patient population is at risk for developing fungal corneal ulcerations? Answer: Immunosuppressed individuals. Corneal Ulcer Patient will have a history of trauma, incomplete closure, or extended contact lens use PE will show oval ulcer with ragged edges, severe conjunctival inflammation Most commonly caused by Staphylococcus, Pseudomonas(contact lens wearers), Streptococcus pneumoniae Treatment is emergent ophthalmology consult

A 35-year-old woman just arrived from vacation where she spent five days skiing in Colorado. She presents to the Emergency Department complaining of bilateral eye pain and tearing. She denies any direct ocular trauma. Her visual fields are intact and no foreign body is identified on examination. There is marked conjunctival erythema, lid swelling, and chemosis. Diffuse punctate lesions are seen on the cornea with fluorescein staining as shown above. Which of the following is the most likely diagnosis? (Not including picture, it is included in many other questions. Also there is a key history element here that gives it away) A Blepharitis B Endophthalmitis C Ultraviolet keratitis D Zoster ophthalmicus

Correct Answer ( C ) Explanation: Ultraviolet keratitis is caused by overexposure to ultraviolet A or B light. It can result from welding, tanning beds, sun lamps, or from light reflected from water or snow (called snow blindness). As a result of fewer clouds and less water vapor in the atmosphere, ultraviolet light is absorbed by the cornea at greater degrees at high altitude. Symptoms of ultraviolet keratitis include significant pain, photophobia, and foreign body sensation. Exam findings include chemosis, conjunctival erythema, lid swelling and punctate lesions on the cornea. It is usually self-limited and heals within 24 hours. However, it is extremely painful and may warrant systemic or topical analgesics and cycloplegics for pain control. Patients should be counselled on the damaging effects of ultraviolet radiation and educated on proper eye protection. Blepharitis (A) is chronic inflammation of the eyelid near the root of the eyelash. Endophthalmitis (B) is an infection of the deep eye structures including the anterior, posterior, and vitreous chambers. It is often due to trauma or recent surgery. They eye appears purulent and may have pus in the anterior chamber. Zoster ophthalmicus (D) is an ocular infection resulting from herpes zoster. Question: What topical antiviral medication is recommended for herpes simplex keratitis? Answer: Trifluridine. Ultraviolet Keratitis Patient will be a skier or welder Complaining of bilateral decreased visual acuity, pain, and redness Fluorescein staining will show multiple punctate lesions Treatment is topical NSAIDs/oral pain medication, +/- antibiotics, +/- cycloplegics

A 55-year-old man presents with complaints of left eye irritation for the past month. He is a landscaper and is often outside. Physical exam reveals a fleshy, triangular growth from the medial canthus toward the cornea as shown above. Which of the following is the most likely diagnosis? FLIP AND LOOK AT THE PICTURE ONLY A Cataract B Chalazion C Pinguecula D Pterygium

Correct Answer ( D ) Explanation: A pterygium is a fleshy, triangular growth that occurs typically as a result of chronic wind, dust, sun, or sand exposure and irritation. These growths may become inflamed and irritated at times or may be asymptomatic. If the growth encroaches far enough onto the cornea, it can cause vision changes. Treatment includes artificial tears and sometimes, topical anti-inflammatory medications. Cataracts (A) do not present as a growth encroaching toward the cornea. Cataracts present with vision changes and an opacification of the lens. Chalazion (B) is a painless growth in the eyelid, not on the conjunctiva or cornea. Pinguecula (C) is a nodular growth on the conjunctiva caused by chronic irritation that in comparison to a pterygium does not encroach on the cornea. Question: What is the definitive treatment of a pterygium? Answer: Surgical excision. Pterygium Patient with a history of excessive sunlight exposure Complaining of something in the eye PE will show triangular shaped growth on the medial side

A 64-year-old woman presents to the emergency department with left eye pain. She was recently diagnosed with Bell's palsy and has had difficulty closing her left eye. You suspect that she has a corneal ulcer. Which of the following physical exam findings is most consistent with this diagnosis? A Dendritic pattern on fluorescein examination B Increased intraocular eye pressure C Peripheral vision loss D White, hazy irregularity of the cornea

Correct Answer ( D ) Explanation: A white hazy irregularity of the cornea can be seen with a corneal ulceration. A corneal ulcer is a bacterial infection that develops secondary to a break in the corneal epithelium. Risk factors for developing a corneal ulcer include incomplete lid closure (e.g. secondary to Bell's palsy) and soft contact lenses use. Symptoms include redness, swelling of the lids, foreign body sensation, and photophobia. Physical exam may reveal a round or an irregular ulceration with a white or hazy base. A secondary iritis can develop and cause ciliary flush or a limbal injection. Treatment includes topical antibiotics and emergent ophthalmologic consultation. Cultures can be obtained of the ocular drainage to guide antibiotic therapy. Cycloplegic drops can be used to help with the pain from the secondary iritis. Dendritic pattern on fluorescein examination (A) signifies an infection with the herpes simplex virus. Increased intraocular eye pressure (B) is associated with glaucoma. Peripheral vision loss (C) is not associated with the corneal ulcer, however, vision loss can occur if the irregularity is located centrally. Question: What is the first line antibiotic for a corneal ulceration? Answer: A topical fluoroquinolone. Corneal Ulcer Patient will have a history of trauma, incomplete closure, or extended contact lens use PE will show oval ulcer with ragged edges, severe conjunctival inflammation Most commonly caused by Staphylococcus, Pseudomonas(contact lens wearers), Streptococcus pneumoniae Treatment is emergent ophthalmology consult

A 61-year-old woman presents to your office with a sudden painless loss of vision in her right eye. Her past medical history includes both hypertension and type 2 diabetes mellitus. Which one of the following would make you suspect retinal vein occlusion as the cause of her sudden visual loss? A An afferent pupillary defect in the contralateral eye B Macular drusen on funduscopic examination C Right eye scleral injection D Tortuous retinal veins on funduscopic examination

Correct Answer ( D ) Explanation: Central retinal vein occlusion (CRVO) is a common vascular disorder of the retina and one of the most common causes of vision loss worldwide. Specifically, it is the second most common cause of blindness from retinal vascular disease after diabetic retinopathy. The signs of retinal vein occlusion typically include sudden painless loss of vision, distortion of vision or dense central scotoma. In some cases, this loss of vision is subtle in character, with intermittent episodes of blurred vision. In other cases, it may be sudden and dramatic. The nonischemic type is often the more subtle of the two, while the ischemic type is prone to the more acute clinical presentations. Tortuous and dilated retinal veins are the most common finding on funduscopic examination. Diabetes mellitus and hypertension are both risk factors for retinal vein occlusion, increasing the likelihood in this patient. Patients also often have multiple macula drusen (B) commonly known as cotton-wool spots, although these are not specific to retinal vein occlusion. Eye redness (C) is not typical and should cause the clinician to suspect an alternate diagnosis. An afferent pupillary defect (A) often occurs on the ipsilateral side. Question: What are two complications of retinal vein occlusion? Answer: Macular edema and neovascularization. Central Retinal Vein Occlusion Patient with a history of HTN or DM Complaining of varying degree of sudden painless monocular vision loss Fundoscopic exam will show "blood and thunder," tortuous, and dilated retinal veins Management includes ophthalmology consultation

A 73-year-old man presents with painless vision loss in the right eye. Which of the following on funduscopic examination is most characteristic of central retinal vein occlusion? A Cherry-red fovea B Pale retina C Papilledema D Retinal hemorrhages

Correct Answer ( D ) Explanation: Central retinal vein occlusion occurs as a result of thrombus formation in the retinal vein. This can occur in the setting of multiple causes including mechanical compression, sluggish circulation, vasculitis, and hypercoagulability. As the central retinal vein is occluded, there is increased resistance to venous flow in the retinal venous system. The impaired bloodflow can increase pressure causing retinal hemorrhages and the classic "blood and thunder" appearance on funduscopic examination. In addition to these findings, the retinal veins may appear dilated and tortuous with macular and optic disc edema (unilateral). Ultimately the pressure increase leads to retinal ischemia and subsequent vision loss. The degree of vision loss depends on how much venous obstruction actually occurs. The cherry red fovea (A) is seen in cases of central retinal artery occlusion. Central retinal artery occlusion essentially causes an ischemic infarct of the retina. In this case, examination reveals a pale retina (B) with the fovea standing out as a cherry-red spot. Papilledema (C) is usually a bilateral physical examination finding where the optic disc is edematous with blurred edges. This is representative of conditions that cause increased intracranial pressure. Question: What is the earliest sign of papilledema on fundoscopic examination? Answer: Loss of normal venous pulsations. Central Retinal Vein Occlusion Patient with a history of HTN or DM Complaining of varying degree of sudden painless monocular vision loss Fundoscopic exam will show "blood and thunder," tortuous, and dilated retinal veins Management includes ophthalmology consultation

A 48-year-old man was hit in the right eye with a softball last night and comes to your office today complaining of new onset of objects floating through his vision along with continuous light flashes and reduced brightness in that eye. Exam reveals full extraoccular movement in all directions. He does complain of mild pain around the eye. Based on these findings, which of the following is the most likely diagnosis? A Corneal abrasion B Lens dislocation C Optic neuritis D Retinal detachment

Correct Answer ( D ) Explanation: Retinal detachment can be observed after blunt eye injury, especially in older adults. Retinal detachment may also occur spontaneously, especially in patients with high myopia. The patient may complain of unilateral blurred vision described as a "curtain coming down" over the eye. There is also reduced overall brightness in the involved eye and continuous light flashes, indicating retinal traction. An increase in previous floaters or the onset of new floaters may also occur. After eye trauma, it is imperative to inspect not only the central portions of the retina, but the peripheral portions as well. This examination should be performed in a dark room after instillation of a short-acting mydriatic agent. Any questionable findings should be referred to an ophthalmologist immediately. Corneal abrasions (A) cause patients to complain of a foreign body sensation in the eye and pain with blinking. It is not associated with visual floaters or flashes of light. Optic Neuritis (C) is associated with a painful loss of vision and loss of color vision. Patients with lens dislocation (B) may complain of decreased vision, edge glare, diplopia, streaks of light, haloes, photosensitivity, and ghost images. An abnormal appearing lens will be seen on slit lamp exam. Question: Traumatic retinoschisis along with intracranial or intraocular hemorrhages in a child under 5 should alert you to what possibility of what condition? Answer: Shaken baby syndrome (Nonaccidental inflicted neurotrauma). Retinal Detachment Patient will be complaining of painless loss of vision, floaters, flashing lights, curtain lowering sensation PE will show retina appears hazy gray with white folds Management is stat ophthalmology consult

An elderly woman underwent cataract surgery last week. Today, she complains of central vision loss and peripheral flashing lights. This patient is likely to emergently require which of the following procedures? A Blepharoplasty B Canaloplasty C Laser assisted in-situ keratomileusis D Pneumatic retinopexy

Correct Answer ( D ) Explanation: Retinal detachment refers to a peeling away of the retina from its underlying supportive connective tissue. Risk factors include prior cataract surgery, head or eye trauma, family history and severe myopia. Considered an emergency, the most common symptoms are acute central vision loss, an acute increase in the numbers of floaters, peripheral vision flashes of light (photopsia), and eye heaviness. Fundoscopic exam with photography is the recommended evaluation when detachment is suspected. 85% of cases improve with one surgical procedure, while the remaining 15% improve with two or more surgical procedures. Common surgeries include cryo therapy and laser photocoagulation, scleral buckle therapy, pneumatic retinopexy and vitrectomy. Blepharoplasty (A) is an eyelid plastic surgery used to remove excessive skin or fat ("eyelift"). Canaloplasty (B) is used to improve trabecular meshwork drainage in a patient with increased intraocular pressure. Laser assisted in-situ keratomileusis (LASIK) (C) is used to correct refractive vision issues, not retinal detachment. Question: Retinal detachment of which area favors a poor prognosis? Answer: The macula. Retinal Detachment Patient will be complaining of painless loss of vision, floaters, flashing lights, curtain lowering sensation PE will show retina appears hazy gray with white folds Management is stat ophthalmology consult

A 46-year-old man presents with left eye redness since this morning as seen in the image above. He denies any trauma, pain, discharge, or vision changes. Which of the following is the most likely diagnosis? (Dont flip as picture has the answer present. This is just some generalized splotchy redness on the cornea) A Conjunctivitis B Hyphema C Retrobulbar hematoma D Spontaneous subconjunctival hemorrhage

Correct Answer ( D ) Explanation: Spontaneous subconjunctival hemorrhage is caused by the rupture of a small subconjunctival vessel that leads to bleeding between the conjunctiva and the sclera. Symptoms include a painless, reddened eye without visual changes. Hemorrhage can occur spontaneously or secondary to forceful coughing, sneezing, or vomiting. Patients using anticoagulants may be at greater risk. Treatment is observation and reassurance, although, sometimes artificial tears may be used to soothe irritated or chemotic tissue. Conjunctivitis (A) is an inflammation of the conjunctiva caused by either a viral or bacterial infection, or an allergen, irritant, or chemical exposure. It is characterized by pain, foreign body or scratchy sensation, watery or purulent discharge, and conjunctival hyperemia. Hyphema (B) is characterized by bleeding into the anterior chamber of the eye usually secondary to blunt trauma. Symptoms of hyphema include eye pain and a sudden decrease in visual acuity that may gradually worsen. Retrobulbar hematoma (C) is usually the result of orbital trauma. Increased pressure on the optic nerve can result in blindness. The condition is associated with proptosis and restriction of extraocular movements. Question: What medications might you discourage your patients from using when they present with subconjunctival hemorrhage? Answer: NSAIDs. Subconjunctival Hemorrhage Patient will be complaining of blood in the eye PE will show blood under conjunctiva and stops at limbus Most commonly caused by coughing, sneezing, minor trauma Treatment is reassurance

Which of the following is most characteristic of acute conjunctivitis? A Ciliary injection B Diminished visual acuity C Pain and photophobia D Serous or clear discharge

Correct Answer ( D ) Explanation: The most common cause of a red eye is acute conjunctivitis, most commonly due to viral infection. Patients often complain of an itchy, irritated eye with serous or clear discharge. Purulent (creamy white or yellow watery) discharge suggests a bacterial cause. Scanty, white, stringy exudate occurs most often with allergic conjunctivitis. It is important to examine both eyes because many patients with conjunctivitis in one eye have signs of early conjunctivitis in the other. Conjunctival injection is characterized by individually visible vessels in the conjunctiva branching from the sclera toward the cornea. Inspect the palpebral conjunctiva carefully with magnification to determine whether lymphoid hyperplasia (cobblestone appearance) exists. The type and quantity of discharge are assessed by pulling down the lower lid. The absence of discharge should prompt investigation to other causes of a red eye. Ciliary injection (A) appears as engorgement of the deep vessels around the limbus. The significance of ciliary injection is that the deep ciliary vessels are involved, indicating a much more serious inflammatory condition of the eye. Diminished visual acuity (B) is not associated with acute conjunctivitis. If present, other serious causes of red eye should be investigated. Pain and photophobia (C) is associated with glaucoma and keratitis. Question: What is the intraocular pressure found in acute conjunctivitis? Answer: Normal (10-20 mmHg). Bacterial Conjunctivitis Patient will be complaining of red/pink eye with discharge usually worse in the morning PE will show purulent (yellow) discharge and crusting Most commonly caused by Staph aureus, Strep pneumoniae, H.Flu.Contact lens wearers: Pseudomonas aeruginosa Treatment is topical antibiotic drops

A 42-year-old man is concerned about redness in the eye. The patient had food poisoning last night and vomited multiple times overnight. Today he saw bleeding in his eye as seen in the image above. What is the appropriate evaluation and treatment? FLIP FOR IMAGE DONT CHEAT - JESUS IS WATCHING A Anterior chamber evaluation B Coagulation studies C Complete blood count D Reassurance

Correct Answer ( D ) Explanation: This patient has a subconjunctival hemorrhage. These hemorrhages occur when a blood vessel ruptures under the surface of the conjunctiva. On physical examination, the hemorrhage is flat and bright red and does not cross the limbus. These hemorrhages may occur spontaneously or as a result of Valsalva maneuvers or trauma. In this case, the patient reports multiple episodes of vomiting over the night preceding recognition of the hemorrhage. The hemorrhages do not cause any long-term sequelae and treatment is aimed at reassurance. A subconjunctival hemorrhage is typically painless without any associated visual impairment or photophobia. If these conditions are present, other diagnoses should be pursued. An anterior chamber evaluation (A) is part of the complete physical examination of the eye using the slit lamp. The anterior chamber may develop inflammatory changes and debris in keratitis. Additionally, subtle hyphemas (collection of blood in the anterior chamber) may be difficult to see without inspection of the anterior chamber. Coagulation studies (B) are indicated in patients on anticoagulation therapy. However, in the routine evaluation of a subconjunctival hemorrhage, these tests are not necessary. A complete blood count (C) to measure platelets is indicated if there is concerned for thrombocytopenia in the patient. This may include patients with other areas of active bleeding, patients on chemotherapy or with marrow suppressing diseases. Question: How long does it take for a subconjunctival hemorrhage to resolve? Answer: 10-14 days. Subconjunctival Hemorrhage Patient will be complaining of blood in the eye PE will show blood under conjunctiva and stops at limbus Most commonly caused by coughing, sneezing, minor trauma Treatment is reassurance

A 32-year-old woman presents with right eye pain and a foreign body sensation. A fluorescein enhancement is shown above. Which of the following is most likely indicated? (Image shows corneal abrasion) A Acyclovir and follow up B Eye patching and follow up C Lid eversion, oral antibiotics and follow up D Lid eversion, topical antibiotics and follow up

Correct Answer ( D ) Explanation: This patient presents with a traumatic corneal abrasion requiring topical antibiotics and follow up. It is also important to perform lid eversion on all patients with corneal abrasion to ensure that the foreign body that caused the abrasion is no longer present. Corneal abrasions are common and cause pain, photophobia, deceased visual acuity and a foreign body sensation. A hallmark of corneal abrasion is the complete or near-complete relief of pain with topical anesthetic drops. Physical examination often reveals conjunctival injection and may show a gross abrasion. Addition of fluorescein and viewing with a cobalt blue light can enhance the abrasion. If the abrasion lies over the visual axis, acuity can be significantly reduced. Treatment involves careful inspection for any retained foreign body including under the eyelid. Antibiotic ointment or drops should be employed and follow up should be scheduled with an ophthalmologist in 24-48 hours. Patients who wear contacts require special consideration. Antibiotic drops in these patients should have pseudomonal coverage and contacts should not be worn until cleared by an ophthalmologist. Acyclovir (A) is required for the treatment of herpes keratitis, which forms a branching pattern defect on the cornea. Eye patching (B) should be avoided. Oral antibiotics (C) are not necessary for simple corneal abrasions. Question: What is Seidel's sign? Answer: Streaming of aqueous humor leaking from the anterior chamber during fluorescein examination secondary to penetration of the cornea. Corneal Abrasion Patient will be complaining of pain, photophobia, tearing, or foreign body sensation Diagnosis is made by fluorescein stain Treatment: Most are self-limiting,anti pseudomonal for contact lens wearers

A 3-day-old male neonate develops bilateral purulent eye discharge. He also exhibits a runny-nose as well as eyelid edema and redness. No other abnormalities on physical examination are noted. Which of the following could have been given as standard prophylaxis to prevent the development of this infant's condition? A Intravenous acyclovir B Intravenous ceftriaxone C Oral erythromycin D Topical erythromycin

Correct Answer ( D ) Explanation: This patient's presentation of bilateral eye discharge on day 3 of life is consistent with gonococcal ophthalmia neonatorum. Topical 0.5% erythromycin applied immediately after birth is effective prevention against development of neonatal gonococcal conjunctivitis. There are 3 common types of neonatal conjunctivitis encountered on board exams: chemical, gonococcal, and chlamydial. They may be easily differentiated based on the time of presentation. Chemical conjunctivitis often occurs within the first 24 hours of life and is typically secondary to the use of certain prophylactic topical ointments applied to newborns to prevent gonococcal conjunctivitis. Gonococcal conjunctivitis occurs 3-5 days after birth and chlamydial conjunctivitis occurs 5-10 days after birth. The US Preventive Services Task Force (USPSTF) issued a recommendation in 2012 stating that all newborns should receive prophylactic ocular topical medication in order to prevent development of gonococcal ophthalmia neonatorum. Acceptable agents include topical erythromycin 0.5%, topical tetracycline 1.0%, silver nitrate, or povidone-iodine 2.5%. However, erythromycin 0.5% ointment is the only drug approved by the FDA for this indication. IV acyclovir (A) is used to treat neonatal herpetic conjunctivitis, which may present with vesicles on the skin surrounding the eyes or other evidence of systemic involvement. IV ceftriaxone (B) is a recommended regimen for the treatment of, not prophylaxis against, neonatal gonococcal conjunctivitis. PO erythromycin (C) may be used to treat chlamydial conjunctivitis; however, it is not recommended for prophylaxis of gonococcal ophthalmia. Artifical tear drops may be useful in the prevention and/or treatment of chemical conjunctivitis, which most commonly manifests within the first 24 hours after birth. Question: What is the most common infectious cause of neonatal conjunctivitis? Answer: Chlamydia. Neisseria Gonorrhea Conjunctivitis Newborns < 5 days old, sexually active adults Hyperpurulent discharge Corneal ulceration/perforation Admission Topical + IV antibiotics

A 35-year-old man presents to the emergency department at 3:00 am complaining of intense bilateral eye pain, redness, and tearing that woke him from sleep fifteen minutes prior to arrival. Eye examination with fluorescein staining of the corneas reveals diffuse punctate corneal lesions as shown above. Further history would likely reveal which of the following? A Allergies to dust mites, tree pollen, pet dander, and cockroaches B Continuous contact lens usage for the past four months C Corrosive chemical exposure D Welding without eye protection earlier in the day

Correct Answer ( D ) Explanation: Ultraviolet keratitis is the result of corneal epithelial damage from exposure to intense UV light. It can be from welding without eye protection, prolonged exposure to sunlight, or using a tanning bed without eye protection. Signs and symptoms include intense eye pain, photophobia, redness, and tearing with diffuse punctate corneal lesions seen with fluorescein staining. Onset of symptoms is typically delayed for several hours. Treatment consists of topical cycloplegics, broad spectrum antibiotic eye ointment, and ophthalmologic follow-up. Allergies to dust mites, tree pollen, pet dander, and cockroaches (A) is a feature of allergic conjunctivitis. Allergic conjunctivitis is characterized by bilateral eye itching and conjunctival injection with a normal fluorescein examination. Continuous contact lens usage for the past four months (B) is a cause of corneal ulcers. Corneal ulcers are epithelial defects typically caused by Pseudomonas aeruginosa in the setting of continuous contact lens usage. They present with pain, redness, photophobia, and tearing, but are usually unilateral and consist of a single corneal lesion. Corrosive chemical exposure (C) can potentially cause diffuse corneal damage, but would not present with a delay in onset of symptoms. Question: What classic corneal finding is seen with herpes simplex keratitis? Answer: Dendritic lesions. Ultraviolet Keratitis Patient will be a skier or welder Complaining of bilateral decreased visual acuity, pain, and redness Fluorescein staining will show multiple punctate lesions Treatment is topical NSAIDs/oral pain medication, +/- antibiotics, +/- cycloplegics

A 75 year-old patient with history of macular degeneration and hypertension presents with complaint of sudden onset of visual loss in the left eye. The patient denies pain. On examination you note a dome-shaped retina and subretinal fluid that shifts with position changes. Which of the following is the most likely diagnosis in this patient? A. Central retinal vein occlusion B. Acute Angle-closure glaucoma c. Acute nongranulomatous anterior uveitis D. Serous retinal detachment

D.

Which of the following clinical findings differentiates periorbital from orbital cellulitis? A. erythema B. fever C. lid edema D. worsening pain with eye movements E. development of a rash on the face

D. Periorbital cellulits is characterized by warmth, redness, swelling, and tenderness over the affected eye, along with conjunctival injection, eyelid swelling, chemosis, and fever. Orbital cellulitis includes all the symptoms of periorbital (preseptal) cellulitis with the addition of ocular pain and limitation of eye movement. Other physical examination findings may include lid edema, proptosis, marked tenderness to the globe, decreased visual acuity, and pupillary paralysis.

A 37 year old male presents with itching and redness at the base of his eyelids. On examination you find scaley patches and red skin at the naso-labial folds. What is the most likely diagnosis? A. Ectropion B. Orbital cellulitus C. Pterygium D. blepharitis with seborrheic dermatitis

D. Question 1 Explanation: A patient with blepharitis will present with eyelid changes that include crusting, scaling, red-rimming of eyelid and eyelash flaking. It is associated with with seborrhea and rosacea.

In a patient with amaurosis fugax (transient loss of vision uni/bil) what is the most appropriate initial diagnostic study? A. Ophthalmoscopy B. Schiotz tonometry C. MR angiography D. Carotid ultrasound

D. Question 1 Explanation: The most common cause of amaurosis fugax is an atherosclerotic plaque in the carotid artery which can be identified with ultrasound.

A 9 year-old patient presents with conjunctivitis after swimming at the local pool. On examination, there is visible lid edema with redness of the palpebral conjunctiva, copious watery discharge, and scanty exudate. The sanitation system of the public pool is through the use of a salt water system; therefore, the possibility of a chemical induced conjunctivitis is almost non-existent. Which of the following should be instituted to prevent the sequalae of the condition A. Ketorolac tromethamine (Acular) B. Dexamethasone ophthalmic C. Naphazoline HCL (Naphcon A) D. Sulfacetamide ophthalmic

D. Question 10 Explanation: One of the most common causes of viral conjunctivitis is adenovirus type 3. Contaminated swimming pools can be source of infection. Topical sulfonamides prevent secondary bacterial infection.

Which of the following is the leading cause of permanent visual loss in a patient over the age of 75? A. Blepharitis B. Cataracts C. Central retinal artery occlusion D. Macular degeneration

D. Question 3 Explanation: Age-related macular degeneration is the leading cause of permanent visual loss in the older population. The exact cause is unknown, but the prevalence increases with each decade over age 50 years.

A patient presents with the complaint of irritation of the left eye one day after gardening. He states "I think there is something in my eye." Which of the following findings is consistent with your suspected diagnosis? A. increased intraocular pressure B. rust ring C. hazy cornea D. fluorescein uptake

D. Question 4 Explanation: Fluorescein dye uptake is diagnostic for corneal abrasion

A 12 year-old presents with complaint of both eyes "watering." He also complains of sinus congestion and sneezing for two weeks. On exam vital signs are T-38°C, P-80/minute, and RR-20/minute. The eyes reveal mild conjunctival injection bilaterally, clear watery discharge, and no matting. Pupils are equal, round, and reactive to light and accommodation. The extraocular movements are intact. The funduscopic exam shows normal disc and vessels. The TMs are normal and the canals are clear. The nasal mucosa is boggy, with clear rhinorrhea. Which of the following is the most helpful pharmacologic agent? A.Artificial tears B. Tobramycin drops C. Erythromycin ointment D. Naphazoline (Naphcon-A) drops (Antihistamine)

D. Question 6 Explanation: Naphazoline is a topical antihistamine that relieves symptoms of allergic conjunctivitis.

A 27-year-old man presents with a foreign body sensation in his right eye. The patient turned on the heat in his car this morning and he felt something fly into his eye. His visual acuity is intact. What is the most appropriate next step in managing this patient? A Emergent ophthalmologic consultation B Obtain an intraocular eye pressure C Order a CT scan of the orbits D Slit lamp examination with fluorescein

D. Slit lamp examination with fluorescein should demonstrate a corneal foreign body if one is present. Corneal foreign bodies are usually superficial and most are easily removed with topical anesthesia. Foreign bodies may be irrigated off of the cornea with normal saline or dislodged with a moistened cotton applicator. If a foreign body is embedded in the cornea, a small gauge needle can be used to remove the foreign body. Metallic foreign bodies can leave rust rings behind in the cornea that can be toxic to the tissue. It is not necessary to remove the rust ring emergently as these patients will need to be followed by an ophthalmologist and an ophthalmic burr can be used to remove the rust ring at the follow-up visit. After foreign body removal, fluorescein stain should be performed. A topical antibiotic should be prescribed and follow-up with ophthalmology should occur within the next 24 to 48 hours. Emergent ophthalmologic consultation (A) is not necessary unless the foreign body is deeply embedded in the cornea or globe perforation is suspected. Obtaining an intraocular eye pressure (B) is typically not necessary when evaluating for suspected corneal foreign body. Ordering a CT scan of the orbits (C) could be useful if the patient is complaining of vision loss or you suspect globe perforation. Question: What is the most concerning complication of not removing a corneal rust ring? Answer: Corneal scarring with associated vision loss. Corneal Abrasion Patient will be complaining of pain, photophobia, tearing, or foreign body sensation Diagnosis is made by fluorescein stain Treatment: Most are self-limiting,anti pseudomonal for contact lens wearers

45-year-old woman presents to the ED with acute painless loss of vision, photophobia associated with a smaller unilateral pupil on the involved side. Which of the following is the most likely diagnosis? A. central retinal vein occlusion (CRVO) B. iritis/uveitis C. retrobulbar hemorrhage or hematoma D. hyphema E. central retinal artery occlusion

E. Question 2 Explanation: Central retinal artery occlusion is characterized by acute visual loss usually attributed to ischemic or thrombus to the major retinal arterial blood supply. Typically, the patient presents with sudden, painless onset of markedly decreased unilateral loss of vision. Physical examination findings include significant decrease in visual acuity, relative afferent pupillary defect (ie, Marcus Gunn pupil), and a pale retina with a red spot that is visible on funduscopic examination. Central retinal vein occlusion (CRVO) is characterized by painless, unilateral vision loss of varying severity, slower onset of decreased vision than with arterial occlusion, retinal hemorrhages, cotton wool spots, and macular edema. Physical examination findings include ciliary flush (ie, circumcorneal perilimbal injection of the episcleritis and scleral vessels) conjunctival injection and cells may be present in the anterior chamber. The pupil on the affected side is often small and irregular. Direct and consensual light reflex will cause pain on the affected side to increase. Retrobulbar hemorrhage is associated with decreased ocular range of motion, decreased vision, ptosis of the lid, and increased pressure in the globe raising intraocular pressure. The high pressure decreases retinal artery perfusion, which results in retinal ischemia. The patient presents with decreased visual acuity, proptosis, and a dilated nonreactive pupil. A hyphema is caused by bleeding from the vasculature of the iris usually precipitated by trauma. Blood is often visualized in the anterior chamber and can be seen via slit lamp evaluation. Symptoms usually consist of pain, photophobia, and decreased vision. Intraocular pressures may increase as well. The major clinical consideration is the potential of reoccurring bleeding. Monocular vision loss with amaurosis fugax THINK THESE - Retinal artery occlusion - Optic neuritis - GCA - May be related to ruptured plaque from ipsilateral carotid artery - Atrial fibrillation

A 28-year-old woman presents with intense pain, tearing, and ocular foreign body sensation in both eyes that began yesterday. She denies any trauma, past medical problems, or contact lens use and reports that she works as ski patrol for a local resort. Her physical examination is significant for bilateral decreased visual acuity, injected conjunctiva, and diffuse punctuate corneal lesions with a discrete superior border. Her pupils are equal, round, and reactive to light. Given this patient's clinical presentation, which of the following is the most likely diagnosis? A. Epidemic keratoconjunctivitis B. Episcleritis C. Iritis D. Optic neuritis E. Ultraviolet keratitis

E. This patient has ultraviolet keratitis, also known as snow blindness. This is a radiation burn that occurs when an individual comes in close contact with an ultraviolet-ray-containing light source. It may be caused by sun lamps, tanning booths, or exposure to intense reflected sunlight, particularly in high-altitude environments. Patients usually present about 6-12 hours post-exposure, complaining of eye pain, blepharospasm, tearing, photophobia, and foreign body sensation. Fluorescein staining reveals superficial punctate epithelial surface irregularities, which usually cover the entire surface of the cornea. This condition is usually referred to as superficial punctate keratitis (SPK). If the patient's eyelid was partially closed during the exposure, a well-demarcated line may distinguish normal from damaged corneal epithelium. Treatment consists of topical NSAIDs or oral pain medication while the use of antibiotics or cycloplegics is controversial. Ophthalmic antibiotics have not been shown to improve healing, but in high risk individuals (chronic contact lens users or immunocompromised individuals) they can be beneficial. Some sources advocate cycloplegics to relieve photophobia, but the pupillary dilation can be distressing to patients. While topical anesthetics may be required during the examination, they should not be prescribed for further use due to an increased risk of corneal ulceration. Epidemic keratoconjunctivitis (EKC) (A) is a highly contagious adenovirus infection, which is distinguished from typical viral conjunctivitis by involvement of the conjunctiva and cornea. A painful keratitis involving the central cornea may develop after one week. Episcleritis (B) is a common, benign inflammatory condition of the episclera and most often occurs in young adults. It is usually idiopathic but may be associated with systemic conditions such as diabetes mellitus, systemic lupus erythematosus, rheumatoid arthritis, and inflammatory bowel disease. Patients typically complain of a foreign body sensation, mild pain, photophobia, and lacrimation. The condition is usually self-limiting. Iritis (C), also known as anterior uveitis, is an inflammation of the anterior portions of the uvea (iris, ciliary body, and choroid). It is often idiopathic but may be associated with systemic diseases such as rheumatoid arthritis, sarcoidosis, reactive arthritis, and tuberculosis. Patients report pain, tearing, and photophobia, and may have decreased visual acuity. Exam typically reveals ciliary flush with a miotic pupil that may be distorted due to the formation of synechiae (iris adheres to either the cornea [anterior synechia] or lens [posterior synechia]). The classic finding on slit lamp exam are cells (sign of inflammation or bleeding) and flare (elevated concentration of plasma proteins from inflamed, leaking intraocular blood vessels) in the anterior chamber. Optic neuritis (D) presents with sudden or rapidly progressing monocular vision loss in patients younger than 50 years. There may be pain on movement of the globe, and the pupillary light response is diminished in the affected eye (afferent pupillary defect). Although optic neuritis is often an isolated condition, it is associated with demyelinating disorders such as multiple sclerosis. Ultraviolet Keratitis Patient will be a skier or welder Complaining of bilateral decreased visual acuity, pain, and redness Fluorescein staining will show multiple punctate lesions Treatment is topical NSAIDs/oral pain medication, +/- antibiotics, +/- cycloplegics

A 44-year-old female presents with persistent right eye irritation for 3 months. The patient complains of excessive tearing and a slight FBS. She denies visual changes, trauma and contact-lens use. Complete ocular exam reveals no abnormality other than her bottom eyelid is turned inward. What is this condition called? A. Ectropion B. Entropion C. Exotropia D. Esotropia

Entropion


Set pelajaran terkait

DV400 Development in Theory and Practice

View Set

Management of Patients with Hematologic Neoplasms

View Set

PrepU Chp 28: Assessment of Hematologic Function and Treatment Modalities

View Set

Origins and Insertions (Abductor Pollicis Longus)

View Set